Exam 1 concepts

Réussis tes devoirs et examens dès maintenant avec Quizwiz!

A charge nurse is reviewing orthostatic hypotension with a group of newly licensed nurses. Which of the following statements should the charge nurse make? "The first step in checking for orthostatic hypotension is obtaining a client's blood pressure while they are standing." "An increase of 5 millimeters of mercury in the diastolic pressure with a position change indicates orthostatic hypotension." "A decrease of 20 millimeters of mercury in the systolic pressure with a position change indicates orthostatic hypotension." "Wait 5 minutes to check the client's blood pressure after each position change."

"A decrease of 20 millimeters of mercury in the systolic pressure with a position change indicates orthostatic hypotension."

A charge nurse is providing an in-service for a group of nurses about cardiac output. Which of the following statements should the nurse include? "Cardiac output is the amount of blood flow through the heart in 1 minute." "Cardiac output is the amount of blood ejected from the atria." "Cardiac output is the ability of the muscle fibers in the ventricles to stretch." "Cardiac output is the resistance of the ventricles to pump blood through the heart."

"Cardiac output is the amount of blood flow through the heart in 1 minute."

A charge nurse is discussing a client's respiratory data with a newly licensed nurse. Which of the following statements should the nurse include? "Clients will exhibit an increase in their respiratory rate after using a bronchodilator." "Count the respiratory rate for 1 minute for clients who have a respiratory infection." "Expect clients who have a brainstem injury to exhibit rapid respirations." "Clients who are experiencing acute pain will have slow, deep respirations."

"Count the respiratory rate for 1 minute for clients who have a respiratory infection."

A charge nurse is reviewing the expected reference range of blood pressure in adult clients with a newly licensed nurse. Which of the following statements should the charge nurse include? "Hypertension is diagnosed with two elevated measurements on two separate occasions." "Successive blood pressure measurements of 126 over 78 is classified as stage I hypertension." "Stage II hypertension is diagnosed when the blood pressure measurement is 132 over 86." "A blood pressure measurement of 176 over 102 is classified as a hypertensive crisis."

"Hypertension is diagnosed with two elevated measurements on two separate occasions."

A nurse in a surgeon's office is providing preoperative teaching for a client who is scheduled for surgery the following week. The client tells the nurse that "I plan to prepare my advanced directives before I come to the hospital." Which of the following statements made by the client should indicate to the nurse and understanding of advance directives? "I'd rather have my brother make decisions for me, but I know it has to be my wife." "I know they won't go ahead with the surgery unless I prepare these forms." "I plan to write that I don't want them to keep me on a breathing machine." "I will get my regular doctor to approve my plan before I hand it in at the hospi

"I plan to write that I don't want them to keep me on a breathing machine."

A nurse is preparing for a presentation on professional identity in nursing. Which of the following statements should the nurse use to describe professional identity? "Professional identity is how one describes themselves as a professional." "Professional identity refers to public image about the role of nurses." "Professional identity is a term describing the belief and acceptance of the standards and values of the profession." "Professional identity begins with licensure and training within a health care setting."

"Professional identity is how one describes themselves as a professional."

A charge nurse is discussing mechanisms of loss of body heat with a newly licensed nurse. Which of the following statements should the nurse include? "Convection is the loss of body heat when a client is in contact with a cooler surface." "Conduction is the loss of body heat when sweat dries from a client's skin." "Evaporation is the loss of body heat when a client is near a current of cool air." "Radiation is the loss of body heat when a client is in close proximity to a cooler surface."

"Radiation is the loss of body heat when a client is in close proximity to a cooler surface."

A nurse is providing teaching about thermoregulation to a group of newly licensed nurses. Which of the following statements should the nurse include in the teaching? "The body increases body temperature through the process known as vasodilation." "The body loses heat through shivering." "The body lowers body temperature through sweating." "The body generates heat through evaporation."

"The body lowers body temperature through sweating."

A nurse is discussing chain of command with a newly licensed nurse. Which of the following statements by the newly licensed nurse indicates an understanding of the discussion? "The use of the chain of command is reserved for natural disasters or acts of terrorism." "The steps for the chain of command consist of notifying my superiors starting from the top to the bottom." "I will wait to document the communication I used while following the chain of command until the end of the process." "The chain of command clarifies the steps I need to take when I have a concern about a client's care."

"The chain of command clarifies the steps I need to take when I have a concern about a client's care."

A nurse is exploring leadership styles of nurse managers as a part of a quality improvement project. Which of the following statements by a nurse manager should the nurse identify as an example of transactional leadership? "You are a smart and highly competent team. I will let you do what you do best." "We've agreed to provide excellent client care and brainstormed the steps we will take to meet our objectives." "I appreciate your suggestion, but each of you will wash your hands before exiting a client's room." "The staff members who complete their online learning courses before the deadline will receive a gift card."

"The staff members who complete their online learning courses before the deadline will receive a gift card."

A nurse manager is providing education to a group of newly licensed nurses about various nursing organizations. Which of the following information should the nurse manager include? - American Nurses Associations code of ethics -National Academy of Medicine - American Academy of Nursing - National Student Nurses Association

- American Nurses Associations code of ethics

A nurse is reviewing historical nursing data & how it has impacted nursing care. The nurse should identify that which of the following nurse pioneers is credited w/ first using evidence based practice in caring for clients? - Clara Barton - Dorthea Dix - Eddie Bernice Johnson - Florence Nightingale

- Florence Nightingale

A nurse is reviewing the Institute of Medicine (IOM) Future of Nursing report to better understand proposed educational requirements of nurses. Which of the following organizations is responsible for publishing this report? - American Nurses Association - National Academy of Medicine - State Board of Nursing - American Association of Colleges of Nursing

- National Academy of Medicine

A nursing student expresses a desire to become more involved w/ the nursing profession on a national level. Which of the following organizations should the student join? - National League for Nursing - National Student Nurses' Association - National Academy of Medicine - International Council of Nurses

- National Student Nurses' Association

A nurse is reviewing the health history of a client during a routine office visit. Which of the following observations by the nurse is an example of the client demonstrating health literacy? - client states they want to improve their current knowledge about their health - client asks for information that will help them improve their eating habits - client states they understand their health care information and treatment - client requests a meeting w/ nurse, physical therapist and provider

- client states they understand their health care information and treatment

A nurse is performing the role of case manager for a client. Which of the following actions demonstrates this nursing role? - coordinating & overseeing the care the client is receiving - helping to develop nursing knowledge for clinical interventions - providing knowledgeable & compassionate care to promote health & address illness - instructing the client on specialized topics such as diabetes care

- coordinating & overseeing the care the client is receiving

A nurse educator is planning a presentation for nurses about the opioid epidemic. Which of the following is a strategy promoted by the U.S. Department of Health and Human Services to address this epidemic? - continue to evaluate to determine if the use of opioids improves on its own - educate nurses and other health care personnel about naloxone - keep the issue private to avoid embarrassment - reduce efforts to explore alternative pain management practices until the opioid epidemic is over

- educate nurses and other health care personnel about naloxone

A nurse is caring for a client who reports experiencing chills and is not well. The nurse informs the client that they will need to have their temperature taken to monitor the manifestations. Which of the following terms describes that the nurse's action is grounded in research? - evidence based practice - competencies - lifelong learning - change agent

- evidence based practice

A nurse is explaining to a newly licensed nurse that caring for clients involves lifelong learning by the nurse. The nurse should include which of the following as examples of learning opportunities for nurses? (select all that apply). - nursing blogs or opinions - online continuing education journal articles - nursing skill refresher opportunities - continuing education presentations - participating in healthcare research

- online continuing education journal articles - nursing skill refresher opportunities - continuing education presentations - participating in healthcare research

A nurse is reviewing the contributions made by various nursing organizations that have impacted the nursing profession. The nurse should identify that which of the following are contributions made by the American Nurses Association (ANA)? (select all that apply). - providing a definition of nursing - lobbying for whistle blower protection - providing objective data to improve the health of society - providing all RN's w/ competency expectations - providing the nursing magazine imprint

- providing a definition of nursing - lobbying for whistle blower protection - providing all RN's w/ competency expectations

A nurse who completed their shift realized they forgot to take a client's vital signs as frequently as prescribed. Which of the following actions should the nurse take to uphold the American Nurses Associations Code of Ethics for nurses w/ interpretive statements? - take the clients vital signs now & document them as being completed 8 hr earlier - document vital signs that are the same as those obtained - make reminder note to be more diligent next day - report this omission to charge nurse and the nurse on the next shift

- report this omission to charge nurse and the nurse on the next shift

A nurse is describing to another nurse how to use social media in client care. Which of the following examples should the nurse include as an acceptable use of social media? - posting a comment on social media about facility staffing shortage - reviewing the use of a fitness application w/ a client - accepting a social media friend request from a client - sharing client information w/ a coworker in a private social media message

- reviewing the use of a fitness application w/ a client

A nurse is caring for a client who asks about healthy choices to lose weight. The nurse shares information from Healthy People 2030 to address the client's questions. Which of the following describes the Healthy People Initiative? - program for health care practitioners to get healthy - social media platform for nurses - set of objectives for improving the health of Americans - rehabilitation program for clients who have substance use disorders

- set of objectives for improving the health of Americans

A nurse in a long-term care facility took a picture of a client and posted it on the internet. This action by the nurse is an example of which of the following? - serving as an advocate for the client - violating the code of ethics for nurses - participating in the Healthy People 2030 - demonstrating collaboration

- violating the code of ethics for nurses

A nurse is discussing culturally competent care w/ another nurse. Which of the following statements should the nurse include? -"use a medical interpreter for a client who does not speak the same language" - "provide the client w/ information in print only so they can have it as a resource" - "provide standard educational materials to all clients for continuity" - "limit communication with the client if there is a language barrier"

-"use a medical interpreter for a client who does not speak the same language"

A nurse is caring for a client who has a range order of hydromorphone 1 to 3 mg IV q4H prn pain score 7 to 10. The client receives 1 mg of hydromorphone at 0900 hours and is still experiencing pain of 10 on a 0 to 10 scale; the client then receives another 1 mg of hydromorphone at 0925 hours. The client reports pain of 7 on a 0 to 10 scale after this administration. If the client needs another dose of hydromorphone, at what time could the client receive a 2 mg dose?

1325 hours or 1:25 p.m. At 0900 hours, the client received 1 mg of hydromorphone. At 0925 hours, the client received another 1 mg, for a total of 2 mg of hydromorphone. Because this total dose was effective for the client, the nurse can give a full dose of 2 mg in 4 hours. 0925 + 0400 = 1325 hours (1:25 p.m.).

a nurse is performing an admission assessment on a client. the nurse determines the client's radio pulse is 68/min And the simultaneous apical pulse is 84/min. what is the client's pulse deficit?

16/min

While collecting data on a client who is immobile, a nurse locates a reddened area of skin on the left scapula. Which of the following actions should the nurse take? 1. reposition the client every 4 hr 2. cover the area with a transparent wound barrier 3. massage areas surrounding the redness 4. wash the area with hot water every 8 hr

2. cover the area with a transparent wound barrier

A nurse is caring for a client who is diagnosed with a urinary tract infection and is prescribed ciprofloxcin (Cipro) 250 mg PO two times daily. The amount available is 100mg/tablet. How many tablets should the nurse administer with each dose?

2.5 tablets

A nurse is providing discharge education to the parents of a preschooler who is prescribes acetaminophen (tylenol) 300 mg every 4 hr as needed. The acetaminophen liquid suspension that has been prescribed provides 120 mg/5mL. How many teaspoons should the nurse teach the parents to administer per dose

2.5 tsp

A nurse is caring for a client who has a prescription for heat therapy for pain in the knee. the nurse should apply heat therapy to the client's need for how long? 60 minutes 20 minutes 30 minutes 45 minutes

20 mins

A nurse is caring for a group of clients on the Pediatric unit. for which of the following clients should the nurse use the FLACC Pain Scale to determine their pain level? (select all that apply) 12-year-old client who had an appendectomy 3-year-old toddler who has a fractured femur 6 day old infant who had surgical repair of a heart defect 14 year old client who has severe cognitive and developmental delays 5-year-old preschooler who was experiencing pain during a Sickle Cell crisis

3-year-old toddler who has a fractured femur 14 year old client who has severe cognitive and developmental delays 5-year-old preschooler who was experiencing pain during a Sickle Cell crisis

A nurse is caring for a client who is receiving intermittent enteral tube feedings and having diarrhea after each feeding. Which of the following actions should the nurse take in an attempt to prevent diarrhea after subsequent feedings? 1. chill formula prior to administration 2. verify feeding tube placement 3. reduce the rate of the feedings 4. place the client supine during feedings

3. reduce the rate of the feedings

A nurse is collecting nutritional data on an older adult client. Which of the following findings is suggestive of a healthy nutritional status? 1. spongy gums that are receding 2. fissures at eyelid corners 3. easily plucked hair 4. deep reddish-colored tongue

4. deep reddish-colored tongue

A nurse is assisting with the preparation of an education program regarding advance directives for newly hired staff. Which of the following information should be included about living wills? 1. living wills require a written prescription from the provider to be legal 2. living wills allow the client to designate a health care proxy 3. living wills ensure hospitals provide emergency care regardless of health coverage 4. living wills detail treatment wishes of the client in the event of terminal illness

4. living wills detail treatment wishes of the client in the event of terminal illness

A nurse is planning care for a group of clients and is reviewing the recent vital signs obtained by an assistive personnel. Which of the following clients should the nurse assess and recheck the vital signs prior to notifying the provider? 16-year-old female: respiratory rate 18/min, SaO2 98% 8-year-old male: respiratory rate 34/min, SaO2 97% 11-year-old male: respiratory rate 28/min, SaO2 99% 3-year-old female: respiratory rate 32/min, SaO2 96%

8-year-old male: respiratory rate 34/min, SaO2 97%

The nurse applicant has completed the NCLEX registration process and has received a confirmed appointment to take the NCLEX. The applicant has experienced an unforeseen circumstance and needs to change the appointment date. The applicant realizes the Authorization to Test (ATT) is valid for approximately which of the following number of days?

90 days: Once a nurse applicant has completed the NCLEX registration and received the ATT email, the applicant has approximately 90 days to take the NCLEX exam.

A nurse is reviewing the vital signs of four clients. The nurse should identify that which of the following clients has a vital sign outside of the expected reference range? A 52-year-old client who has a fever due to a wound infection and a pulse rate of 100/min A 76-year-old client who reports moderate pain and has a respiratory rate of 20/min A 46-year-old client who is postoperative following a hysterectomy and has an SaO2 of 95% A 23-year-old client who runs marathons and has a blood pressure of 82/54 mm Hg

A 23-year-old client who runs marathons and has a blood pressure of 82/54 mm Hg

A nurse on a pediatric unit is reviewing the medical records for a group of clients. Which of the following clients has a vital sign outside the expected reference range and requires intervention? A 1-month-old infant who has a respiratory rate of 58/min A 3-year-old preschooler who has an apical pulse rate of 144/min An 8-year-old child who has a respiratory rate of 25/min An 18-month-old toddler who has an apical pulse rate of 120/min

A 3-year-old preschooler who has an apical pulse rate of 144/min

A nurse is reviewing the recent vital signs of a group of clients. Which of the following clients should the nurse see first? A 45-year-old client who is postoperative and has a BP of 130/82 mm Hg A 28-year-old client who runs marathons and has a heart rate of 54/min A 52-year-old client who has an SaO2 of 92% A 78-year-old client who has a temperature of 35.9° C (96.6° F)

A 52-year-old client who has an SaO2 of 92%

A nurse is caring for a client who is experiencing tachypnea due to an exacerbation of asthma. Which of the following medications should the nurse anticipate administering? A nicotine product An opioid antagonist An antihypertensive A bronchodilator

A bronchodilator

A nurse is evaluating the effectiveness of interventions provided to four clients who have unexpected findings for vital signs. Which of the following findings requires follow up? A client has an 8 mm Hg difference in systolic BP when moving from a sitting to a standing position. A client has a radial pulse of +4 bilateral. An older adult client has a tympanic temperature of 35.9° C (96.6° F). A newborn has a respiratory rate of 56/min while sleeping.

A client has a radial pulse of +4 bilateral.

A charge nurse in a clinic is preparing an in-service about blood pressure measurements for a group of staff members. Which of the following information should the nurse include? A client is diagnosed with an elevated blood pressure when the measurement is greater than 130/80 mm Hg. A client is experiencing a hypertensive crisis when their blood pressure is greater than 150/90 mm Hg. A client who has a blood pressure of 128/86 mm Hg has stage I hypertension. A client who has a blood pressure of 162/102 mm Hg has stage II hypertension.

A client who has a blood pressure of 162/102 mm Hg has stage II hypertension.

A nurse is assisting with client triage at the scene of a mass casualty event. Which of the following clients should the nurse recommend for transport first? A client who reports a possible sprained wrist and is walking around A client who has an open forearm fracture without visible drainage A client who has a respiratory rate of 6/min and no pupil response A client who has an abdominal wound that is actively bleeding

A client who has an abdominal wound that is actively bleeding

A nurse working on a medical-surgical unit is caring for a group of clients. Which of the following clients' vital signs should the nurse identify is outside the expected reference range and notify the provider? A client who has an apical pulse rate of 120/min A client who has a blood pressure of 100/74 mm Hg A client who has an apical pulse rate of 84/min A client who has a blood pressure of 110/68 mm Hg

A client who has an apical pulse rate of 120/min

A nurse has received change-of-shift report on four clients. Which of the following clients should the nurse plan to see first? A client who is schedules for an abdominal ultrasound A client who has audible wheezing during respirations A client who has audible wheezing during respirations A client who requests their routine pain medication

A client who has audible wheezing during respirations

A nurse is teaching a group of assistive personnel (AP) about techniques used to obtain BP. For which of the following clients should the nurse to instruct the AP to obtain an electronic BP measurement? A client who has a BP lower than the expected reference range A school-age child A client recovering from extensive abdominal surgery A client who has stabilized BP measurements

A client who has stabilized BP measurements

A nurse is caring for a group of clients on a medical-surgical unit. For which of the following client care needs should the nurse initiate a referral for a social worker? (Select all that apply) A client who has terminal cancer requests hospice care in the home A client asks about community resources available for older adults A client states, "I would like to have my child baptized before surgery" A client requests an electric wheelchair for use after discharge A client states, "I do not understand how to use a nebulizer"

A client who has terminal cancer requests hospice care in the home A client asks about community resources available for older adults A client requests an electric wheelchair for use after discharge

A nurse is planning care for a group of clients. For which of the following clients should the nurse direct an assistive personnel (AP) to obtain a rectal temperature? A toddler who has diarrhea A client who is 1 day postoperative following a hemorrhoidectomy and receiving pain medications via PCA pump An infant who is receiving intravenous fluids A client who is diaphoretic and frequently chewing ice to relieve dry mouth

A client who is diaphoretic and frequently chewing ice to relieve dry mouth

A nurse is assisting with mass-casualty triage of clients following a gas explosion. Which of the following clients should the nurse recommend for priority treatment? 1. a client who is unable to walk, has burns on both legs, nad reports hurting too much to move 2. a client with a metal rod penetrating the forearm 3. small laceration on forehead and is walking around aimlessly 4. client with no spontaneous breathing)

A client who is unable to walk, has burns on both legs, nad reports hurting too much to move

A nurse is reviewing the medical records of four clients. Which of the following clients should the nurse identify as the priority for care? A client who received digoxin and has a heart rate of 48/min Client who received pain medication and has a respiratory rate of 14/min Client who has a UTI and temperature of 100.2 Client who has anemia and BP of 118/78

A client who received digoxin and has a heart rate of 48/min

A nurse at a provider's office is reviewing the records of several clients. Which of the following clients should the nurse recommend as the priority for treatment? A client who reports new chest pain A client who has a history of hypertension and requires a yearly checkup A client who reports increased joint stiffness due to arthritis A client who has diabetes mellitus and needs dietary instruction

A client who reports new chest pain

In what order should a clients be cared for using Maslow's Hierarchy?

A client who requests assistance to the bathroom so they can void A client who is concerned about being able to safely use crutches after discharge A client who is upset because none of their friends from school has been in to visit A client who is upset about their body image because of hair loss A client who requests to be discharged because they need to focus on career goals

A nurse is planning care for a group of clients and is delegating to the assistive personnel (AP) to take the clients' vital signs. For which of the following clients should the nurse obtain the vital signs rather than the AP? A client who just received the fourth dose of an antibiotic for an infection A client who has heart failure and is scheduled for discharge later in the day A client who is 24 hr postoperative and is visiting with friends A client who was recently admitted and reports chest pain

A client who was recently admitted and reports chest pain

A nurse is instructing a group of newly licensed nurses about how to know and what to expect when ethical dilemmas arise. Which of the following situations should the newly licensed nurse identify as an ethical dilemma? A nurse of a medical surgical unit demonstrates signs of chemical impairment A nurse overhears another nurse telling an older adult client that if he doesn't stay in bed, she will have to apply restraints A family has conflicting feelings about the initiation of enteral tube feedings for their father, who is terminally ill A client who is terminally ill hesitates to name their partner on their durable power of attorney form

A family has conflicting feelings about the initiation of enteral tube feedings for their father, who is terminally ill

A nurse is planning a presentation about shared governance. Which of the following examples should the nurse plan to include? A supervisor at a home health agency makes a unilateral decision to cut staff. Two assistant directors at an ambulatory care center write the policy manuals for the center. A group of staff nurses in a clinic work together to recommend new equipment purchases for the unit. Nurses on a medical-surgical floor follow instructions made by the charge nurse.

A group of staff nurses in a clinic work together to recommend new equipment purchases for the unit.

How does a nurse demonstrate responsibility and accountability by performing hand hygiene before seeing each client?

A nurse is responsible for washing their hands (action or task) between each client, but they are accountable to infection control practices, quality client care, safe practices, and standards of practice, and they are answerable for the consequences and/or outcomes. Responsibility and accountability are connected. The key difference is that the nurse is responsible for completing an action or a task and accountable in the manner in which they are done to something or someone, such as the organization, the NPA, and/or scope of practice. Accountability is being completely liable for the decision, action, and outcome.

Give a nursing example that meets the definition of evidence-based practice?

A nurse searches the recent research literature to revise a nursing procedure.

A nurse is evaluating the effectiveness of interventions used for clients who had alterations in vital signs. Which of the following clients' vital signs indicate that interventions were effective? (Select all that apply.) A preschooler who was exhibiting tachypnea 2 hr postoperative and now has a respiratory rate of 26/min An older adult client who had bradycardia while sleeping and now has an apical pulse rate of 66/min A young adult who had hypotension after receiving an opioid analgesic and now has a blood pressure of 98/68 mm Hg A toddler who was febrile 2 hr ago due to a viral infection and has a temporal temperature of 38.2° C (100.8° F) An adult client who had tachycardia 1 hr ago due to postoperative pain and has an apical pulse rate of 106/min

A preschooler who was exhibiting tachypnea 2 hr postoperative and now has a respiratory rate of 26/min An older adult client who had bradycardia while sleeping and now has an apical pulse rate of 66/min A young adult who had hypotension after receiving an opioid analgesic and now has a blood pressure of 98/68 mm Hg

A nurse is preparing an in-service about peripheral pulses for a group of staff nurses. Which of the following information should the nurse include? A pulse strength of +4 indicates that the pulse is of normal strength upon palpation. A femoral pulse that is bounding upon palpation is an expected finding in a young adult. A pulse strength of +1 indicates that the pulse is weak or diminished upon palpation. A pedal pulse that is weak upon palpation is an expected finding in an older adult.

A pulse strength of +1 indicates that the pulse is weak or diminished upon palpation.

A nurse is caring for a group of clients. Which of the following clients is experiencing an alteration in their respiratory rate that requires intervention? An adolescent who has a respiratory rate of 20/min An older adult who has a respiratory rate of 16/min An infant who has a respiratory rate of 52/min A school-age child who has a respiratory rate of 14/min

A school-age child who has a respiratory rate of 14/min

A nurse is preparing an in-service for a group of newly hired assistive personnel (AP) about body temperature. Which of the following information should the nurse include? Wait 5 min after a client has consumed a hot drink to obtain an oral temperature. Place a tape or patch thermometer over a client's scapula. A tympanic thermometer reflects a client's body surface temperature. A temporal probe thermometer uses infrared scanning to determine a client's temperature.

A temporal probe thermometer uses infrared scanning to determine a client's temperature.

A nurse is reviewing the vital signs for a group of clients obtained by an assistive personnel. The nurse should identify that which of the following clients requires a follow-up assessment due to bradycardia? school-age child who has an apical pulse rate of 78/min A young adult who has a radial pulse rate of 56/min An adolescent who has a radial pulse rate of 76/min An older adult who has an apical pulse rate of 62/min

A young adult who has a radial pulse rate of 56/min

A nurse is reviewing the vital signs for a group of clients. Which of the following clients should the nurse identify as exhibiting tachycardia? An infant who has an apical pulse rate of 132/min A preschooler who has an apical pulse rate of 108/min A young adult who has an apical pulse rate of 104/min An older adult who has an apical pulse rate of 96/min

A young adult who has an apical pulse rate of 104/min

A nurse is evaluating the effectiveness of interventions used to address clients' vital signs that were outside of the expected reference ranges. Which of the following findings indicates the intervention was effective? An older adult client who has pneumonia and a respiratory rate of 26/min after a position change An adolescent who is postoperative and has an SaO2 of 93% after receiving an opioid analgesic A young adult who is experiencing an asthma attack and has a blood pressure of 116/72 mm Hg after using an inhaler An older adult client who has an infection and a pulse rate of 110/min after using relaxation techniques

A young adult who is experiencing an asthma attack and has a blood pressure of 116/72 mm Hg after using an inhaler

A nurse is caring for a client who is experiencing unexpected manifestations with several body systems. Which of the following priority setting frameworks should the nurse use to prioritize client assessment? Acute vs chronic ABCDE Least restrictive/least invasive Survival potential

ABCDE

A nurse is reviewing the contributions of various nursing organizations that provide support in caring for clients. The nurse should identify that which of the following agencies provides The Nursing Scope and Standards of Practice? - American Nurses Association (ANA) - American Academy of Nursing - National StudentNurses Association (NSNA) - National Academy of Medicine

ANA

A nurse returns to a client's room 25 minutes after administering morphine 2 mg IV. How should the nurse evaluate the effectiveness of the intervention?

Ask the client to rate the pain level

A nurse follows the six rights of medication administration when caring for a client. Which of the following concepts is the nurse demonstrating? Laissez-faire Advocacy Professional identity Accountability

Accountability

a nurse is working with the information technology department of his facility to establish a protocol regarding security mechanisms that will protect the electronic health records of clients. which of the following could result in a violation of client confidentiality Placement of a computer in restricted areas Installation of firewall software on each computer Ability of staff to access EHR of clients throughout the facility Occurrence of an automatic log-off after a period of inactivity

Ability of staff to access EHR of clients throughout the facility

a nurse is caring for a client who has osteoarthritis and is considering treatment with acupuncture. which of the following is acceptable for the nurse to include in discussion with the client Acupuncture is loose regulated by the federal government Acupuncture has been discredited by scientific research Acupuncture is thought to be effective only as a placebo Acupuncture has been proven to reduce pain and increase function

Acupuncture has been proven to reduce pain and increase function

A nurse is discussing leadership styles with a newly licensed nurse. The nurse should include that which of the following is a characteristic of situational leadership? Follows leadership directives exactly as stated Controls situations by paying attention to details Adjusts quickly to different circumstances Refuses to allow staff to make exceptions to established guidelines

Adjusts quickly to different circumstances

A client is admitted to the orthopedic unit at 0845 with a fractured pelvis. The client reports sharp pain of 10/10 in the "hip" that is described as "unbearable whenever I move." The nurse reviews the client's medication administration record (MAR) and observes that the client has a prescription for morphine 2 mg IV every 3 hours as needed for pain. The last dose of morphine was administered at 0430 this morning. What intervention should the nurse take?

Administer morphine. The nurse should identify that the client is experiencing severe pain. According to the client's MAR, it has been 4 hours and 15 minutes since the client received morphine. The nurse reviews the prescription and observes that morphine can be administered every 3 hours as needed for pain. Therefore, the nurse should administer morphine 2 mg IV to the client for pain

What are the advantages of becoming involved in the NSNA?

Advantages of involvement in the NSNA include generating excitement about the profession; connecting to other nurses across the nation; opportunities for leadership and political experiences; the ability to have papers published in NSNA's magazine; generating confidence; gaining experience working with nursing leaders; subscription to Imprint magazine; discounts on study tools, clothes, and conferences; awards programs; and scholarship opportunities.

The nurse administered oxygen to the client via nasal cannula as prescribed by the provider and continued gathering information about the client. The nurse noted the client's oxygen saturation to be at 94% within a few minutes after the initiation of oxygen therapy. When the nurse asked if the oxygen had helped the client's breathing, the client stated, "I feel much better; I feel like I'm getting enough air now." The client's respiratory rate has decreased to 16/min, the BP is 124/80 mmHg, and the pulse is 72/min. The nurse reviews the client's medical history provided by the client's adult child. Which client need should the nurse identify as the second priority?

After ensuring that the client's oxygenation needs have been met, the nurse should identify the client's safety as the next priority. According to Maslow's hierarchy of needs, once all physiological needs have been met, the need for safety becomes the primary goal. The nurse should investigate information regarding the client's fall history to prevent further falls. The client's grief and anxiety are higher-level needs that the nurse will address later.

A nurse is obtaining vital signs for a group of clients. Which of the following findings requires intervention? A 17-year-old who has a respiratory rate of 16/min A young adult who has a pulse rate of 98/min An 11-year-old child who has a respiratory rate of 34/min An older adult who has a pulse rate of 62/min

An 11-year-old child who has a respiratory rate of 34/min

A nurse is reviewing the vital signs for a group of clients to determine the effectiveness of interventions. Which of the following findings indicates an intervention was effective? An adult client who received medication for pain 30 min ago now has a respiratory rate of 18/min. A school-age child who received two units of packed red blood cells now has a BP of 76/54 mm Hg. A toddler who received an antibiotic injection now has a heart rate of 148/min while sleeping in their parent's arms. An older adult client who received an antipyretic medication 1 hr ago now has a temperature of 38.7° C (101.6° F).

An adult client who received medication for pain 30 min ago now has a respiratory rate of 18/min.

A nurse is discussing factors affecting blood pressure with an assistive personnel. What types of factors should the nurse identify as potential causes for an increase in a client's blood pressure?

Anxiety, fear, use of nicotine, obesity

A nurse is caring for a group of clients. Which of the following actions by the nurse demonstrates professionalism? Asks a client for their name, date of birth, and scans their medical identification bracelet before administering medications Determines that a client's rash is a response to being overly stressed about a new diagnosis Suggests a client's family member translate medical information for a client who speaks a different language than the nurse Documents that a client's medication was administered when retrieving the medication from the pharmacy

Asks a client for their name, date of birth, and scans their medical identification bracelet before administering medications

A nurse and urgent care clinic is auscultating the lungs of a client who reports a cough and shortness of breath. Which of the following steps of the nursing process is the nurse using? Evaluation implementation analysis Assessment

Assessment

In what order should an RN perform the steps of the nursing process?

Assessment, analysis, planning, implementation, evaluation

A nurse in an urgent care clinic is caring for a client who has bronchitis with thick pulmonary secretions. The client's oxygen saturation level is 90% on room air. Which of the following actions should the nurse take first? A) Initiate oxygen therapy B) encourage an increase in oral fluids C) provide room humidification D) Assist client to cough effectively

Assist client to cough effectively

A nurse is caring for a client who has asthma. What action should the nurse identify as the priority?

Auscultate lung sounds

A nurse is caring for a client who decides not to have surgery despite significant blockages of the coronary arteries. The nurse understands that this client's choice is an example of which of the following ethical principles? Fidelity Autonomy Justice Nonmaleficence

Autonomy

A charge nurse is planning to discuss factors that can influence the clinical decision making process in client care with a newly licensed nurse. Which of the following factors should the charge nurse include? Available resources awareness of client status Appropriate delegation Cost of client care support from other staff

Available resources, Awareness of client status Support from other staff

A nurse is caring for a client who reports dizziness when standing up. The client's blood pressure after lying supine for 15 minutes is 136/86 mm Hg in the left arm. What finding would indicate the client is experiencing orthostatic hypotension?

B/P 114/72 mm Hg, left arm, immediately after standing

A nurse is assessing a client using the ABCDE approach. The nurse has already assessed the client's airway and breathing status. Which of the following assessments should the nurse perform next? Body temperature Abdominal contour Blood pressure Skin integrity

BP

A charge nurse is evaluating a newly licensed nurse's documentation of vital signs for several clients. Which of the following documentation should the charge nurse identify as being incomplete? Radial pulse regular at 84/min Respirations observed as even, nonlabored at 20/min with client in supine position BP 124/82 mm Hg, lying in bed Temporal temperature 36.9° C (98.4° F)

BP 124/82 mm Hg, lying in bed

A nurse is acquainting a group of newly licensed nurses with the roles of the health care team they will encounter in a medical-surgical unit. When providing examples of the types of tasks certified nursing assistants can perform, which of the following client activities should the nurse include? (select all that apply) Bathing Ambulating Toileting Determining pain level Measuring vital signs

Bathing Ambulating Toileting Measuring vital signs

A nurse offers pain medication to a client who is postoperative prior to ambulation. The nurse understands that this aspect of care delivery is an example of which of the following ethical principles? Fidelity Autonomy Justice Beneficence

Beneficence

A nurse is discussing the physiology of blood pressure with a group of assistive personnel. Which of the following information should the nurse include? Diastolic blood pressure reflects the pressure exerted during contraction of the heart. Blood pressure is measured and documented in millimeters of mercury. Blood pressure decreases when the blood viscosity increases. Systolic blood pressure reflects the pressure when the heart is relaxed.

Blood pressure is measured and documented in millimeters of mercury.

A nurse is teaching a group of newly licensed nurses about vital sign measurements. Which of the following factors should the nurse include in the teaching? Anxiety can cause a decrease in respiratory rate. Body temperature is typically lower in older adults. Caffeine can cause a temporary decrease in pulse rate in adolescents. Blood pressure slightly decreases immediately following the use of nicotine.

Body temperature is typically lower in older adults.

A nurse is preparing information for a change-of-shift report. Which of the following information should the nurse include in the report? Input and output for the shift Blood pressure from the previous day Bone scan scheduled for today Medication routine from the medication administration record

Bone scan scheduled for today

What are some responsibilities of a nurse manager?

Budgeting, scheduling, resolving conflicts, client safety

what tasks can and cannot be delegated to an AP?

Can be delegated: measurement of pulse and urine, collection of stool sample, ambulation to the bathroom Cannot be delegated: wound care of a new wound, assessment of respiratory system, assessment of urine, administration of eye drops

A nurse is discussing nursing roles with a newly licensed nurse. The nurse should include that which of the following is a role of the nurse? (Select all that apply.) Diagnostician Caregiver Advocate Change agent Dispenser of medications

Caregiver Advocate Change agent

a nurse is caring for an older adult client who has an allergy to sulfa, is taking valproic acid for a seizure disorder and has been newly diagnosed with osteoarthritis. the client states, "i keep seeing commercials on TV for celebrex and i want to try it and see if it will help my pain." upon reviewing of scientific evidence the nurse should inform the client of which of the following Celecobix is contraindicated in clients taking valproic acid Celecobix is contraindicated in older adults Celecobix is contraindicated in clients with a seizure disorder Celecobix is contraindicated in clients with an allergy to sulfonamide

Celecobix is contraindicated in clients with an allergy to sulfonamide

A charge nurse uses a transformational leadership style when working with other staff on the unit. Which of the following actions by the charge nurse demonstrates transformational leadership? Tells assistive personnel if the morning care for a group of clients is completed by noon they will receive a free lunch Challenges a nurse to investigate evidence-based approaches to reduce the client fall rate on the unit Provides a nurse with the written protocol for changing a dressing and encourages them to work independently Adjusts direction to nursing staff based on the nurses' experience and client needs

Challenges a nurse to investigate evidence-based approaches to reduce the client fall rate on the unit

A nurse working on the cardiac unit hears an alarm and finds one of the heart monitor screens at the nurse's station is displaying a straight line, indicating a client is in cardiac arrest. Which of the following actions should the nurse take first? A) Check on the client B) unlock the crash cart C) begin cardiopulmonary resuscitation D) announce a code

Check on the client

A nurse is discussing responsibility with a newly licensed nurse. Which of the following actions should the nurse include as an example of responsibility? Informs a social worker that a client will require financial assistance with prescriptions at discharge Supports a client's decision to stop chemotherapy Checks a client to determine how well a pain medication worked Ensures a client understands their treatment options

Checks a client to determine how well a pain medication worked

what are some challenges of assessing pain in children

Children may deny pain to avoid IM injection or bad tasting oral medicine

A nurse is discussing the Model of Professionalism with another nurse. Which of the following concepts should the nurse include as an outer process? Trustworthiness Clear communication Self-awareness Capacity for reflection

Clear communication

What are some influencing factors of pain?

Client risk factors Client trends Anxiety Type of pain medication used

A nurse has received change-of-shift report for a group of clients. Which of the following clients should the nurse plan to see first? Client who is receiving a blood transfusion and reports urticaria Client who has back pain and is requesting a muscle relaxant medication Client who has an ankle sprain and requests toileting assistance Client who has chronic migraines and reports a headache

Client who is receiving a blood transfusion and reports urticaria

A nurse is reviewing methods created to assist nurses in using evidence-based practice. Which of the following is a NCSBN model that can assist the nurse is critical thinking and decision making? Clinical judgment Critical thinking clinical reasoning SMART goal

Clinical judgment

Define the following ANA Professional Standards with the correct definition: code of ethics for nurses, nursing scope and standards of practice, ANA position statement, ANA principles for nursing practice

Code of Ethics for Nurses: identifies behaviors and practices nurses should abide by to provide quality, ethical care Nursing Scope and Standards of Practice: identifies where, when, why, and how nursing occurs and the actions that all nurses are expected to perform competently ANA Position Statement: provides and explanation and suggested course of action when there is a change in practice, practice concerns, or a topic related to nursing that needs to be addressed ANA Principles for Nursing Practice: provides practice information regarding issues such as social media, staffing, and pay for nursing staff

The PN plans to return to the client's bedside and explain that promethazine is usually effective in relieving nausea, and that this medication is administered rectally. The PN enters the client's room and asks the client, "Have you ever received a suppository before?"; the PN then explains the procedure before its administration. The PN assists the client to a comfortable position. The client asks for a cool towel to place on the forehead to assist in relieving nausea, saying this has helped in the past. Write the steps of the PN nursing process used and explain how the PN addressed each one in this scenario.

Collect data: The PN asks if the client has previously had a suppository. Plan: The PN plans to reinforce teaching on the medication usage and administration. Implement: The PN explains the suppository procedure, assists the client into a comfortable position, and provides a cool towel per the client's request.

A nurse is caring for a client who is in an acute care facility. The nurse should recognize that the clients care requires clinical reason when it is complicated by which of the following factors? Complex clinical situations ongoing client family concerns Cost of health care Decreased need for advanced healthcare practitioner intervention Availability of computerized medical records

Complex clinical situations Ongoing client family concerns

A nurse is caring for a client who is confused and trying to remove their peripheral IV. Using the least restrictive/least invasive priority setting framework, which of the following actions should the nurse take first? Apply a soft limb restraint to the client's wrists Administer an antianxiety medication to the client intramuscularly Request a prescription for a central venous catheter Cover the IV site with an elastic bandage.

Cover the IV site with an elastic bandage.

A nurse is reviewing the concept of critical thinking with a newly licensed nurse. Which of the following statements should the nurse make? Critical thinking is the foundation for clinical decision making Critical thinking takes into consideration nursing, scientific, and technological knowledge and client situations critical thinking is the visible or observed outcome while using evidence-based practice critical thinking is necessary for the nurse to collect objective client data

Critical thinking is the foundation for clinical decision making

List the four steps of the PN nursing process in order for review.

Data collection, planning, intervention, evaluation

A nurse is discussing leadership styles with a newly licensed nurse. Which of the following characteristics should the nurse identify as describing a bureaucratic leader? (Select all that apply.) Decisions are controlled by policies Enforces the rules Inflexible to creative suggestions Uses a variety of strategies based upon the circumstances Motivates staff with the use of rewards or punishments

Decisions are controlled by policies Enforces the rules Inflexible to creative suggestions

A nurse is caring for a client who has hypotension. Which of the following factors should the nurse identify as a contributing factor to the client's condition? Decrease in contractility Increase in blood viscosity Decrease in respiratory rate Increase in preload

Decrease in contractility

A nurse is caring for a client who has been wheezing. The nurse asked an assistive personnel do use a stethoscope and listen to the clients lung sounds to determine if their wheezing has improved. This is an example of which of the following concepts? Delegation of the right circumstance Delegation of the wrong task delegation to the right person delegation of the wrong time

Delegation of the wrong task

a nurse is reinforcing teaching about performing suctioning to a client who is being discharged following a trachesostomy. which of the following behaviors by the client best indicate to the nurse that teaching has been effective Self-reporting the ability to perform the procedure Answering appropriately when questioned orally Responding accurately on a written exam Demonstrating independent performance of the procedure

Demonstrating independent performance of the procedure

a nurse in a local clinic is caring for a female client who is 35 years old. which of the following screenings should the nurse recommend to the client? 1. Mammogram every year to detect breast cancer 2. Colonoscopy every 10 years to detect colon cancer 3. Dermatology evaluation every 3 years to detect skin cancer 4. Complete eye exam every year to detect eye disorders

Dermatology evaluation every 3 years to detect skin cancer

a nurse discovers that a client who is diagnosed with dementia received the wrong medication. which of the following should be the nurse's first action Information the nurse manager Determine the client's condition Notify the provider Complete an incident report

Determine the client's condition

A nurse is advocating for a client who is seeking treatment for an eating disorder. In which order should the nurse take the following steps? (Move the steps into the box on the right, placing them in the order of performance. Use all the steps.) Encourage the client to communicate their preferences during the team meeting to establish the steps of treatment Determine the client's level of understanding regarding their eating disorder Review the client's weight prior to each treatment session Assist the client in establishing goals for treatment

Determine the client's level of understanding regarding their eating disorder Assist the client in establishing goals for treatment Encourage the client to communicate their preferences during the team meeting to establish the steps of treatment Review the client's weight prior to each treatment session

a nurse is caring for a client following a bronchoscopy. which of the following findings requires immediate intervention Painful swallowing Hoarse voice Difficulty breathing Blood-tinged sputum

Difficulty breathing

A nurse is conducting therapeutic medication monitoring on four clients. Which of the findings should be immediately reported to the provider? A) Lithium carbonate 0.8 mmol/L B) Digoxin 3.0 ng/mL C) Peak serum gentamicin 6 mcg/mL D) Magnesium sulfate 4 mEq/L

Digoxin 3.0 ng/mL

If the client reports that the pain is 4/10 and has been relieved to a tolerable or acceptable level, what should the nurse do with this information?

Document it in the client's medical record. Clients' subjective and objective data both should be recorded in the medical record to show evaluation of the intervention. The evaluation may also be reported to the supervisor, if the situation warrants.

what are some contributions Florence Nightingale made to nursing that are still used today?

Documentation, data collection, hand hygiene, evidence-based practice

A nurse manager is observing a newly licensed nurse provide care for a client. Which of the following actions by the newly licensed nurse should the manager identify as unprofessional behavior? (Select all that apply.) Documents in advance to save time Stays logged into the client's electronic health record while obtaining supplies from the supply room Contacts a client on social media to become friends Informs a provider that a requested action is outside the nurse's scope of practice Provides a certified medical translator for a client who speaks a different language than the nurse

Documents in advance to save time Stays logged into the client's electronic health record while obtaining supplies from the supply room Contacts a client on social media to become friends

A nurse is evaluating the effectiveness of interventions provided to a client who has an SaO2 below the expected reference range. Which of the following manifestations requires follow up by the nurse? Eupnea Dyspnea Heart rate of 84/min SaO2 of 96%

Dyspnea

Name the four stages of hypertension, including the blood pressure values that define each stage.

Elevated: when the systolic pressure is 120 to 129 mm Hg and the diastolic pressure is less than 80 mm Hg Stage I hypertension: when the systolic pressure is 130 to 139 mm Hg, or the diastolic pressure is 80 to 89 mm Hg Stage II hypertension: when the systolic pressure is 140 mm Hg or greater or the diastolic pressure is 90 mm Hg or greater Hypertensive crisis: when the systolic pressure is greater than 180 mm Hg and/or the diastolic pressure is greater than 120 mm Hg

A nurse is assessing a client who has orthostatic hypotension. Which of the following actions should the nurse take? Encourage the client to change positions slowly. Restrict the client's oral intake of fluids. Encourage the client to take a short walk. Discontinue IV fluids.

Encourage the client to change positions slowly.

A nurse is planning care for a client who is experiencing tachycardia. Which of the following interventions should the nurse plan to include? Instruct the client to increase exercise. Instruct the client to consume no more than four caffeinated beverages per day. Encourage the client to practice relaxation techniques each day. Encourage the client to engage in pattern paced breathing by panting.

Encourage the client to practice relaxation techniques each day.

A nurse is caring for a client who has a heart rate of 118/min. Which of the following actions should the nurse take to improve the client's heart rate? Encourage the client to reduce intake of caffeinated soft drinks. Inform the client to ambulate in the hallway for 10 min prior to taking vital signs. Increase the room temperature and add blankets to warm the client. Withhold the client's antianxiety medication.

Encourage the client to reduce intake of caffeinated soft drinks.

A nurse asked the client to write the current level of pain using a scale of 0 to 10 after administering a pain medication 30 minutes ago. Which of the following steps of the nursing process is the nurse performing? Implementation analysis planning Evaluation

Evaluation

A nurse is discussing the FLACC scale with a newly licensed nurse period which of the following categories should the nurse include? ( select all that apply) Face Legs Alert Circulation consolability

Face Legs consolability

A nurse is caring for a competent adult client who tells the nurse, "I am leaving the hospital this morning whether the doctor discharges me or not." The nurse believes that this is not in the client's best interest and prepares to administer a PRN sedative medication the client has not requested along with the scheduled morning medication. Which of the following types of tort is the nurse about to commit? Assault False imprisonment Negligence Breach of confidentiality

False imprisonment This is considered a chemical constraint

When using a blood pressure cuff, what factors can lead to a false low or high

False low: cuff is too high, nicotine False high: cuff is too loose/wrong size, unsupported arm

A nurse manager is discussing the HIPAA Privacy Rule with a group of newly hired nurses during orientation. Which of the following information should the nurse manager include? (select all that apply) A single electronic records password is provided for nurses on the same unit Family members should provide a code prior to receiving client health information Communication of client information can occur at the nurses station A client can request a copy of their medical record A nurse can photocopy a client's medical record for transfer to another facility

Family members should provide a code prior to receiving client health information Communication of client information can occur at the nurses station A client can request a copy of their medical record A nurse can photocopy a client's medical record for transfer to another facility

A nurse is preparing an in-service about factors affecting respiratory rate for a group of assistive personnel. Which of the following information should the nurse include? Anxiety can decrease a client's respiratory rate. Opioid analgesics can increase a client's respiratory rate. Pain can decrease a client's respiratory rate. Fever can increase a client's respiratory rate.

Fever can increase a client's respiratory rate.

A charge nurse is overseeing an assistive personnel (AP) who has been assigned care for a group of clients. Which of the following actions by the charge nurse demonstrates accountability? Assigns the AP to provide care for clients who are in rooms next to each other Files an incident report when the AP performs a task that is outside their scope of practice Promises to write a letter of recommendation for the AP if they complete all their assigned tasks Discusses advancing their education with the AP

Files an incident report when the AP performs a task that is outside their scope of practice

A nurse is assessing a client who is nonverbal for the presence of pain. which of the following findings indicate an increased level of discomfort? ( select all that apply) Grimacing Restlessness elevated temperature increased diaphoresis Bradycardia

Grimacing Restlessness increased diaphoresis

A nurse is discussing shared governance with a newly licensed nurse. Which of the following components should the nurse include in the discussion? (Select all that apply.) Guides decisions toward accountability Improves client outcomes Assures money is being used for its intended purpose Promotes professional development Monitors activities of directors

Guides decisions toward accountability Improves client outcomes Promotes professional development

a nurse is reinforcing teaching to a client who has aphasia. which of the following actions by the nurse is appropriate when communication with the client Raising her voice level when speaking to the client Asking the client open-ended questions Clarifying client statements with family as needed Having the client use eye blinks to indicate yes or no

Having the client use eye blinks to indicate yes or no

What is the correct order for the chain of command, from highest to lowest.

Hospital supervisor - provider - Charge nurse - Registered nurse - Practical nurse

A nurse is teaching a group of nurses and asks which of the NLN and integrating competencies is defined as having respect for diversity, holistic care, client centered approach, and client advocacy. Which of the following competencies does this describe? Human flourishing Nursing judgment Patient-centered care Spirit of inquiry

Human flourishing

A nurse is caring for a client who is diagnosed with rheumatoid arthritis and is prescribed dexamethasone (Prednisone). Which of the following indicates the client is experiencing an adverse effect of the medication? 1. Hypomagnesemia 2. Hyperglycemia 3. Hyponatremia 4.hyperkalemia

Hyperglycemia

A nurse is caring for a client who has a new diagnosis of cancer. The client states, "I don't understand what my treatment options are." Which of the following statements by the nurse demonstrates advocacy? I will contact your provider to review your treatment options Why don't you understand your options? Your provider thinks you need an operation You will need to cut living costs so you can pay for the treatment

I will contact your provider to review your treatment options

A nurse is discussing the nursing process with a newly licensed nurse period which of the following statements by the newly licensed nurse should the nurse identify as appropriate for the planning step of the nursing process? I will determine the most important client problems that we should address. I will review the past medical history on the client's record to get more information. I will carry out the new prescriptions from the provider. I will ask the client if they're nausea has resolved.

I will determine the most important client problems that we should address.

a nurse manager is reviewing with nurses on the unit in the care of a client who has had a seizure period which of the following statements by a nurse requires further instructions? I will place the client on their side I will go to the nurses station for assistance I will note the time that the seizure begins I will prepare to insert an airway

I will go to the nurses station for assistance

A nurse is reviewing discharge instructions for a client who has a prescription for morphine oral solution 10 to 20 mg every 4 hours prn. which of the following statements by the client indicates an understanding of the instructions? I can use the morphine as needed as long as I don't take it more than six times a day I will use my household teaspoon to measure the correct amount of morphine I will monitor for high blood pressure while taking the morphine I will keep the morphine bottle locked in a cabinet in my kitchen

I will keep the morphine bottle locked in a cabinet in my kitchen

A nurse is providing education about a new prescription for nitroglycerin (NitroQuick) to a client who is diagnosed with angina. Which of the following statements by the client indicates a need for further teaching? 1. I'll make sure that the medication container is kept tightly sealed 2. I'm lucky I have a prescription plan that allows me to buy pills in bulk quantities 3. I'll keep my pills in the medicine cabinet when I'm home 4. I'll go to the emergency room if my chest pain doesn't go away

I'm lucky I have a prescription plan that allows me to buy pills in bulk quantities.

a nurse is reviewing the documentation of a newly licensed nurse. which of the following actions by the newly licensed nurse while documenting requires the nurse preceptor to intervene 1. Including a client's nurses' note that an incident report was completed after a medication error 2. Drawing horizontal lines through blank spaces life in the nurses' notes followed by a signature 3. Refusing to chart the vital signs taken by another nurse on a client's graphic flow chart 4. Documenting the provider was contacted to clarify a questionable prescription

Including a client's nurses' note that an incident report was completed after a medication error

A nurse is caring for a client who has an increase in cardiac afterload. Which of the following findings should the nurse expect Increase in blood pressure Increase in respiratory rate Decrease in cardiac output Decrease in preload

Increase in blood pressure

A nurse is caring for a client who has an increase in cardiac output. Which of the following findings should the nurse expect? Increase in blood pressure Decrease in respiratory rate Decrease in heart rate Increase in stroke volume

Increase in blood pressure

A charge nurse is preparing to discuss critical thinking skills with a group of newly licensed nurses. Which of the following skills should the nurse plan to include in the discussion? Inference Creativity inductive reasoning Implementation Inspection

Inference Creativity inductive reasoning

A nurse is providing care to a client who has an apical pulse rate of 54/min and is experiencing dizziness. Which of the following is the nurse's priority action? Teach the client how to take their pulse so they can keep the provider informed of variations. Inform the client to ask for assistance with getting out of bed. Educate the client on medications, including therapeutic effects and potential adverse effects. Ensure the client has been taking medications as prescribed.

Inform the client to ask for assistance with getting out of bed.

a nurse is reinforcing teaching about HIV with a group of HS students. which of the following information is appropriate for the nurse to include Medications will eliminate HIV in most clients Adolescents are at a lower risk for developing HIV Initial HIV symptoms are often similar to the flu Using condoms ensures the prevention of HIV during sex

Initial HIV symptoms are often similar to the flu

what requirements are necessary to apply to initial licensure, licensure renewal, and both?

Initial licensure: verification of graduation from an approved prelicensure nursing education program, completion of background of the NCLEX exam, confirmation of successful completion of NCLEX exam Renewal: completion of continuing education hours (in some states), confirmation of practice hours or activities (in some states) Both: disclosure of disciplinary action taken against any professional license, reporting of any substance misuse within the previous 5 years

A nurse is caring for a client who has a heart rate of 120/min. Which of the following actions should the nurse take? Instruct the client to bear down like they are having a bowel movement. Offer the client hot caffeinated tea to drink early in the morning. Hold the client's thyroid medication. Encourage the client to take a warm shower.

Instruct the client to bear down like they are having a bowel movement.

give examples of intrinsic and extrinsic risk factors

Intrinsic risk factors: age, hormones, ethnicity, genetics Extrinsic risk factors: smoking, exercise, caffeine, sodium

A nurse is instructing a group of newly licensed nurses about the responsibilities organ donation and procurement involve. When the nurse explains that all clients waiting for a kidney transplant have to meet the same qualifications, the newly licensed nurses should understand that this aspect of care delivery is an example of which of the following ethical principles? Fidelity Autonomy Justice Beneficence

Justice

A nurse is teaching staff about the ethical principle of justice and how it relates to pain management for clients. which of the following statements should the nurse make? Justice allows the client the freedom of choice Justice allows the client the opportunity to be treated fairly Justice is causing no harm to the client Justice is doing good for the client

Justice allows the client the opportunity to be treated fairly

Match the following contributions to the name of the nursing leader who made them

Nightingale: data collection, started nursing schools Clara Barton: formalized nursing education, founded the American Red Cross Dorothea Dix: mental health awareness, indigenous rights Eddie Bernice Johnson: U.S. Congressional representative, created opportunities for minorities Ildaura Murillo-Rohde: founded the National Association of Hispanic Nurses

A nurse is preparing a poster describing leadership styles for a unit in-service. Which of the following terms should the nurse use to describe a nurse manager who exerts little control over the decisions made by the charge nurse? Bureaucratic Transactional Transformational Laissez-faire

Laissez-faire

Give an example of the following nursing roles: leader, researcher, case manager, educator

Leader: inspires and motivates a group toward achieving a common goal or cause Researcher: seeks factual evidence-based information for nursing practice Case manager: communicates and coordinates care for the client with all team members Educator: teaches nursing students

A nurse is evaluating the effectiveness of interventions provided to a client who was admitted for decreased circulation. Which of the following findings requires further intervention by the nurse? Pulse deficit of 0 Left radial pulse is nonpalpable Peripheral pulse +2 bilateral Brachial pulses are symmetrical

Left radial pulse is nonpalpable

a nurse on the coronary care unit is caring for a client who was transferred from the medical floor after experiencing myocardial infarction. after the client is stabilized, she asks the nurse why she had to be transferred to a unit where her family will be unable to stay with her ally the time. which of the following responses is appropriate I know this must be frightening, but you're going to be fine Let's talk for a minute about your concerns You were transferred because it is in your best interest Why do you feel a family member should be with you?

Let's talk for a minute about your concerns

A nurse is reviewing the licensing laws in the state of residency. Which of the following statements should the nurse use to define licensure? Licensure increases state revenue Licensure limits the number of practicing nurses Licensure protects the health and welfare of the public Licensure protects a nurse from a malpractice lawsuit

Licensure protects the health and welfare of the public

what position should you put a client in who is here for a vaginal examination. Lithotomy Dorsal recumbent Prone Lateral recumbent

Lithotomy

a nurse is caring for a client who is diagnosed with active pulmonary tuberculosis and is taking isoniazid and ethambutol. which of the following manifestations reported by the client necessitate the discontinuation of ehthambutol Loss of color discrimination Nausea and vomiting Red-orange discoloration to body fluids Edema of feet and hands

Loss of color discrimination

A nurse is preparing a poster about professional accountability for an in-service. Which of the following examples should the nurse include? (Select all that apply.) Informs a client's family that the assistive personnel (AP) failed to get the client out of bed during the day Lowers a client's bed to the lowest position after assisting the client to lie down Administers pain medication to a client 30 min before the client is scheduled for physical therapy Provides instructions to assistive personnel (AP) on how to provide culturally competent care to a client Documents a dressing change on a client's wound prior to completing it

Lowers a client's bed to the lowest position after assisting the client to lie down Administers pain medication to a client 30 min before the client is scheduled for physical therapy Provides instructions to assistive personnel (AP) on how to provide culturally competent care to a client

A nurse is caring for a client who is about to undergo an elective surgical procedure. The nurse should take which of the following actions regarding informed consent? (select all that apply) Make sure the surgeon obtained the client's consent Witness the client's signature on the consent form Explain the risks and benefits of the procedure This is the surgeon's responsibility Describe the consequences of choosing not to have the surgery This is the surgeon's responsibility Tell the client about alternatives to having the surgery This is the surgeon's responsibility

Make sure the surgeon obtained the client's consent Witness the client's signature on the consent form

A nurse is discussing occurrences that require completion of an incident report with newly licensed nurses. Which of the following should the nurse include in the teaching? (select all that apply) Medication error Needlesticks Conflict with provider and nursing staff Omission of prescription Missed specimen collection of a prescribed laboratory test

Medication error Needlesticks Omission of prescription

Give an example of something that has a positive effect on nurse staffing shortages?

More nurses furthering their education

A nurses caring for a client who has a urinary track infection. The client is disoriented and found wandering on another unit. Which of the following actions should the nurse take first? A) Ensure all 4 side rails are up. B) Administer a prescribed sedative. C) Place the client in soft wrist restraints D) Move the client to a room near the nurses' station

Move the client to a room near the nurses' station

What are the requirements for becoming a member of the NSNA?

NSNA membership is open to any nursing student or pre-nursing student in a state-approved program who is preparing for licensure as a registered nurse.

A nurse is caring for a client who is post-operative following abdominal surgery and has a morphine PCA pump. which of the following medications should the nurse ensures available in case the client developed respiratory depression? Naloxone Lidocaine Prednisone Amitriptyline

Naloxone

A nurse is caring for a client who has severe pain and repeatedly asks for pain medication. the nurse is busy and forgets to assess the client's pain and administer prescribed pain medication which of the following can the nurse be charged with? Malpractice Negligence Nonmaleficence beneficence

Negligence

give examples or nociceptive vs neuropathic pain

Nociceptive: broken rib, back pain Neuropathic: sciatic pain, below the knee amputation, trigeminal neuralgia

A nurse questions a medication prescription as too extreme in light of the client's advanced age and unstable status. The nurse understands that this action is an example of which of the following ethical principles? Fidelity Autonomy Justice Nonmaleficence

Nonmaleficence

A nurse is preparing a presentation on the regulation of nursing practice. Place the hierarchy of the regulation of practice in the correct order beginning with the highest level (step one) and proceeding to the lowest level (step four). (Move the steps into the box on the right, placing them in the order of hierarchy. Use all the steps.) Organization policies and procedures Individual self-regulation Nurse regulatory board Nurse practice act

Nurse regulatory board Nurse practice act Organization policies and procedures Individual self-regulation

give some examples of both objective and subjective symptoms and observations of a patient

Objective: crackles in the posterior left lung Systolic heart murmur 20 mL of amber-colored urine Raised red rash on left arm Cool and clammy skin Subjective: Dull pain in right knee Nausea for 2 days Report of difficulty breathing

A nurse obtains a client's electronic blood pressure reading of 188/96 mm Hg. Which of the following actions should the nurse take next? Provide client teaching regarding medications to control blood pressure. Notify the provider of the client's blood pressure reading. Provide client education on measures to decrease blood pressure. Obtain a manual blood pressure reading from the client.

Obtain a manual blood pressure reading from the client.

A goal for a client who has difficulty with self-feeding due to rheumatoid arthritis is to use adaptive devices. The nurse caring for the client should initiate a referral to which of the following members of the interprofessional care team? Social worker Certified nursing assistant Registered dietitian Occupational therapist

Occupational therapist

identify if the medications are opioids, nonopioids, or adjuvants: morphine, acetaminophen, fentanyl, carbamazepine, codeine, gabapentin,

Opioid: morphine, fentanyl, codeine Nonopioid: acetaminophen Adjuvant: gabapentin, carbamazepine

A nurse is preparing an in-service about vital signs for a group of newly hired assistive personnel. Which of the following information should the nurse include about measuring body temperature? Tympanic temperature can be affected by environmental temperature. Temporal temperature is inaccurate in children under 3 years of age. Axillary temperature reflects rapid changes in a client's core body temperature. Oral temperature is easily accessible despite a client's position.

Oral temperature is easily accessible despite a client's position.

A nurse is discussing oxygen saturation with a client. Which of the following information should the nurse include? Oxygen saturation is determined by the amount of oxygen bound to white blood cells. Oxygen saturation reflects the amount of oxygen being delivered to body tissues. The expected reference range for oxygen saturation is 90% to 100%. A capillary refill time of less than 5 seconds ensures a reliable oxygen saturation measurement.

Oxygen saturation reflects the amount of oxygen being delivered to body tissues.

A nurse is admitting a client who has hypertension. Using the nursing process, which of the following actions should the nurse take Develop nursing diagnoses Perform a physical assessment Administer prescribed medications Develop goals and outcomes

Perform a physical assessment

Give an example of each category of Maslow's hierarchy of needs

Physiological Needs: air, food, sleep, clothing, health, shelter Safety: employment, resources, health, and property Love and Belonging: social relationships, family, and friends Esteem needs: ability, achievement, and respect from others Self actualization: reaching one's potential, self-fulfillment

A nurse is prioritizing care for a client. Identify the priority order of client needs using Maslow's Hierarchy of Needs. (Move the levels into the box on the right, placing them in the order of priority. Use all the levels.) Physiological, Self-esteem Safety, Self-actualization Love/Belonging,

Physiological, Safety, Love/Belonging, Self-Esteem, Self-actualization

A nurse is caring for a client who is having difficulty breathing. Which of the following actions should the nurse take first? A) Place O2 at 2 L per nasal canula on the client B) Place the client in the orthopneic position C) Perform chest percussion D) perform nasotracheal suction

Place the client in the orthopneic position

A nurse is reinforcing discharge teaching to a new mother regarding sudden infant death syndrome (SIDS). Which of the following is the highest priority to include in the instructions? A) Place the infant in a supine position when sleeping B) place the infant on a firm mattress when sleeping C) avoid covering the infant with loose bedding while sleeping D) avoid leaving stuffed animals in the crib with the sleeping infant

Place the infant in a supine position when sleeping

A nurse is assessing a 3-month-old infant during a well-child visit. Which of the following actions should the nurse take when assessing the apical pulse? Count the number of beats heard in 15 seconds and multiply by 4. Notify the provider if the apical pulse rate is greater than 110/min. Place the stethoscope over the 4th intercostal space to the left of the sternum. Auscultate the apical pulse for an S4 heart sound.

Place the stethoscope over the 4th intercostal space to the left of the sternum.

a nurse discovers a small paper fire in a trash can in a client's bathroom. the client has been taken to safety and the alarm has been activated. which of the following actions should the nurse take? open the windows in the client's room to allow smoke to escape obtain a class C fire extinguisher extinguish the fire remove all electrical equipment from the client's room Place wet towels along the base of the door to the client's room

Place wet towels along the base of the door to the client's room

A nurse is developing a goal for a client to ambulate with assistance at least once by the end of the shift. The nurse should identify that this is an example of which of the following steps of the nursing process Evaluation Implementation Analysis Planning

Planning

A nurse is reviewing the lab results for four clients. The client with which of the following values requires immediate intervention? A) Cholesterol 220 mg/dL B) Platelets 95,000 mm^3 C) BUN 20 mg/dL D) Potassium 3.5 mEq/L

Platelets 95,000 mm^3

a nurse is caring for a client who has been admitted to the medical unit with vomiting and possible dehydration Blood glucose 150 mg/dL Potassium 2.5 mEq/L Total protein 5.2 g/dL Urine specific gravity 1.040

Potassium 2.5 mEq/L

A nurse is assisting with preparation of a teaching program about healthy nutrition for a group of clients who are tactile learners. What kind of activity should be included as a learning strategy in the program

Prepare a health meal to serve at the end of class

A nurse holds a single state nursing license and is preparing to move to another state. For the nurse to apply for reciprocity, which of the following actions should the nurse take? (select all that apply) Present a current and valid nursing license Submit an application Pay the fees for new license Retake the NCLEX Reapply for specialty nursing certifications held in their previous home state

Present a current and valid nursing license Submit an application Pay the fees for new license

A nurse is teaching a newly licensed nurse about professional competence. Which of the following examples should the nurse include in the teaching? Documenting client care before it is completed Reviewing the charts of clients not assigned to the nurse Presenting a summary of an evidence-based journal Performing a surgical procedure under the direction of a surgeon

Presenting a summary of an evidence-based journal

a nurse is caring for an older adult client who was admitted 3 days ago with fractured ribs bilaterally and is suspected of being abused by his caregivers. which of the following should be the nurse's priority goal Support the client's relationship with his caregivers Encourage the client to express his feelings Determine who is responsible for the abuse Protect the client from further abuse

Protect the client from further abuse

A nurse is planning care for a client who has hypertension. Which of the following interventions should the nurse include in the plan? (Select all that apply.) Provide the client with low-sodium meals and snacks. Encourage the client to participate in physical activity each day. Instruct the client in the use of relaxation techniques. Inform the client of the importance of abstaining from using products that contain nicotine. Anticipate a prescription for a 1L IV fluid bolus.

Provide the client with low-sodium meals and snacks. Encourage the client to participate in physical activity each day. Instruct the client in the use of relaxation techniques. Inform the client of the importance of abstaining from using products that contain nicotine.

A client who is postoperative following knee arthroplasty is concerned about the adverse effects of the medication prescribed for pain management. Which of the following members of the interpersonal care team can assist the client in understanding the medication's effects? (select all that apply) Provider Certified nursing assistant Pharmacist Registered nurse Respiratory therapist

Provider Pharmacist Registered nurse

A nurse is reviewing blood flow through the heart with a group of assistive personnel. The nurse should identify that blood flows to which of the following parts of the heart as it leaves the right ventricle? Tricuspid valve Pulmonary artery Right atrium Vena cava

Pulmonary artery

A nurse is caring for a recently admitted client and as part of the plan of care, two nurses obtained simultaneous pulse rates. The client's auscultated apical pulse was 106/min and the palpated radial pulse was 93/min. The nurse should document the findings as which of the following? ​​Pulse deficit less than 10 Radial pulse irregular Apical pulse greater than radial Pulse deficit of 13/min

Pulse deficit of 13/min

A nurse is reviewing documentation of vital signs by a newly licensed nurse for an assigned client. Which of the following entries in the chart requires follow up by the nurse? BP 130/82 mm Hg left arm, lying. Client reports experiencing postoperative pain as 7 on a scale of 0 to 10. Prescribed analgesic administered and will re-evaluate BP in 30 min. Pulse rate 116/min, left radial, standing, immediately following 10 min of ambulating in hall. SaO2 93% left index finger, client sleeping, nasal O2 dislodged. Nasal O2 readjusted and SaO2 increased to 95%. Respiratory rate 18/min via observation, client sitting in chair.

Pulse rate 116/min, left radial, standing, immediately following 10 min of ambulating in hall.

A charge nurse is teaching a group of assistive personnel (AP) about the importance of documenting accurate vital signs. Which of the following information should the charge nurse include in the teaching? Record vital signs at the end of each shift. Recording vital signs provides critical information regarding a client's condition. Obtaining and documenting baseline vital signs is the responsibility of the AP. It is not necessary to record electronic blood pressure measurements.

Recording vital signs provides critical information regarding a client's condition.

A nurse is receiving a provider's prescription by telephone for morphine for a client who is reporting moderate to severe pain. Which of the following nursing actions are appropriate? (select all that apply) Repeat the details of the prescription back to the provider Have another nurse listen to the telephone prescription Obtain the provider's signature on the prescription within 24 hours Decline the verbal prescription because it is not an emergency situation Tell the charge nurse that the provider has prescribed morphine by telephone

Repeat the details of the prescription back to the provider Have another nurse listen to the telephone prescription Obtain the provider's signature on the prescription within 24 hours

A nurse has noticed several occasions in the past week when another nurse on the unit seemed drowsy and unable to focus on the issue at hand. Today, the nurse was found asleep in a chair in the break room not during a break. Which of the following actions should the nurse take? Alert the American Nurses Association Fill out an incident report Report the observations to the nurse manager on the unit Leave the nurse alone to sleep

Report the observations to the nurse manager on the unit

A charge nurse is reviewing the steps of the nursing process with the group of nurses. which of the following data should the charge nurse identify as objective data? ( select all that apply) Respiratory rate is 22/minute with even, unlabored respirations. the client's partner states they said "they hurt after walking about 10 minutes". the client's pain rating is 3 on a scale of 0 to 10. the client skin is pink, warm, and dry. the assistive personnel reports that the client walked with a limp

Respiratory rate is 22/minute with even, unlabored respirations. the client skin is pink, warm, and dry. the assistive personnel reports that the client walked with a limp

A nurse receives a prescription for an antibiotic for a client who has cellulitis. The nurse checks the client's medical record, discovers that the client is allergic to the antibiotic, and calls the provider to request a prescription for a different antibiotic. Which of the following critical thinking attitudes did the nurse demonstrate? Fairness Responsibility Risk-taking Creativity

Responsibility

A charge nurse is discussing the physiology of the heart with a newly licensed nurse. Which of the following anatomical sites should the newly licensed nurse identify as the pacemaker of the heart? Atrioventricular (AV) node Left ventricle Sinoatrial (SA) node Right ventricle

SA node

A charge nurse is reviewing documentation of vital signs by a newly licensed nurse. Which of the following pieces of documentation is correct? Pulse 52/min Respiratory rate 24 SaO2 97% right index finger, room air Blood pressure 132/86 mm Hg

SaO2 97% right index finger, room air

Give examples of nursing prioritization with the supporting priority-setting framework

Safety and risk reduction: turning off the IV antibiotic of a client experiencing an allergic reaction before administering an antihistamine Acute vs chronic: caring for a client who is experiencing medication toxicity before a client who has back pain from an injury three years ago Least restrictive/least invasive: assisting an AP to sit with a client who is confused and pulling at their IV before applying wrist restraints Survival potential: recommending treatment during a mass casualty incident to a client who has severe but treatable bleeding before a client who has minor abrasions Unstable vs stable: notifying the provider about an elevated potassium level before a hemoglobin level that is within the expected reference range Maslow's hierarchy: offering fluids to a client who has dehydration before assisting them to call a family member

what is the order of how to manually take a blood pressure

Select the correct size cuff based on the client's arm size Position the client's arm at the level of the heart, ensuring it is supported. Apply the cuff snugly to the client's upper arm. Place the bell or diaphragm of the stethoscope over the brachial artery. Inflate the bladder of the cuff to 30 mm Hg above the client's baseline systolic pressure. Auscultate the Korotkoff sounds while deflating the cuff. Document the measurement in the client's medical record.

A charge nurse is reviewing the technique for obtaining SaO2 with a group of newly hired nurses. Identify the order of the steps the nurse should include. (Move the steps into the box on the right, placing them in the order of performance. Use all the steps.) Wait 15 seconds and observe the SaO2 percentage displayed on the pulse oximeter Apply the sensor probe on the chosen site Select the site for obtaining measurement Confirm the pulse rate displayed on the oximeter by palpating the radial pulse

Select the site for obtaining measurement Apply the sensor probe on the chosen site Confirm the pulse rate displayed on the oximeter by palpating the radial pulse Wait 15 seconds and observe the SaO2 percentage displayed on the pulse oximeter

A nurse is caring for a client who reports feeling inferior and states that they are not good enough. The nurse should recognize that these feelings fall under which of the following categories of Maslow's Hierarchy of Needs? Love and belonging Self-actualization Safety Self-esteem

Self-esteem

What are some ways a nurse act as a change agent?

Serve on the quality improvement committee Provide the nurse manager with information on a more efficient cardiac monitor Introduce a new process to discharge clients

a nurse is caring for a client who has nausea and a prescription for promethazine 25 mg IM. which of the following is appropriate when preparing a medication for administration from an ampule Use a filter needle to administer Expel air bubbles back into the ampule Set the ampule on a flat surface to withdraw the promethazine Break the ampule toward the body

Set the ampule on a flat surface to withdraw the promethazine

A charge nurse is talking with a newly licensed nurse and is reviewing nursing interventions that do not require a provider's prescription. which of the following interventions should the charge nurse include? ( select all that apply) Writing a prescription for morphine sulfate as needed for pain Inserting a NG tube to relieve gastric distention Showing a client how to use progressive muscle relaxation Performing a daily bath after the evening meal Repositioning a client every 2 hrs to reduce pressure injury risk

Showing a client how to use progressive muscle relaxation Performing a daily bath after the evening meal Repositioning a client every 2 hrs to reduce pressure injury risk

A nurse is caring for a client who asks about factors that could cause their pulse rate to increase. Which of the following factors should the nurse include in their response? Hypothermia Smoking Sleeping Aging

Smoking

A nurse is reviewing the scope of nursing practice. Which of the following is the purpose of the nursing scope of practice? Provides recommendations for nursing practice Provides competencies for the nurse to achieve before receiving initial licensure Serves as a contractual document between the nurse and their employer Specifies the actions a nurse can perform

Specifies the actions a nurse can perform

a nurse is caring for a client who had a cerebrovascular accident and is having difficulty swallowing. which of the following health care professional should attend next interdisciplinary team meeting to address this complication Speech pathologist Occupational therapist Social worker Respiratory therapist

Speech pathologist

A client who had a cerebrovascular accident has persistent problems with dysphagia. The nurse caring for the client should initiate a referral with which of the following members of the interprofessional care team? Social worker Certified nursing assistant Occupational therapist Speech-language pathology

Speech-language pathology

A nurse is providing client care using the nursing process. The nurse should identify that which of the following standards or guidelines includes this criterion? Standards of care Clinical practice guidelines Standards of practice Standard to plan nursing care

Standards of practice

A nurse is providing education to a group of newly licensed nurses regarding the development of an identity as a nurse as well as behavioral characteristics associated with nursing. Which of the following components of practice addresses the identity and characteristics of nursing? Standard to plan nursing care Standards of professional performance Standards of care Clinical practice guidelines

Standards of professional performance

A nurse has accepted a position at a hospital in the state where they live. The nurse should identify that which of the following regulates the nurse's actions when they begin working? - American Nurses Association (ANA) code of ethics for nurses with interpretive statements - state nurse practice act - national student nurses association (NSNA) - national league for nursing

State NPA

a nurse is reinforcing teaching by demonstrating deep breathing and coughing exercises to a client who is scheduled for abdominal surgery. for which of the following responses by the client should the nurse postpone teaching? States that pain is an 8 on a scale of 0 to 10 States her partner should be given the information Expresses concern about the exercises causing pain when performed after surgery Expresses uncertainty about the benefits of the exercises

States that pain is an 8 on a scale of 0 to 10

A charge nurse is reviewing factors that can affect a client's perception of pain with a newly licensed nurse period which of the following should the charge nurse include? ( select all that apply) Stress dietary practices Culture social support disease severity

Stress Culture social support disease severity

A nurse receives a license renewal notification from the board of nursing. The nurse should identify that a license renewal requires the disclosure of which of the following? Medical history Substance use within the past 5 years Certifications received Current salary

Substance use within the past 5 years

A newly licensed nurse is reviewing information about the National Council of state board of nursing transition to practice program. Which of the following factors should the nurse expect? TTP programs increase job retention. TTP programs decrease the ability to provide safe client care. TTP programs increase stress in the workplace. TTP programs decrease competency.

TTP programs increase job retention.

In the video, the nurse states that they are both accountable and responsible. Give two examples of actions you can take now to demonstrate accountability and responsibility as a nursing student.

Taking ownership of decisions and ultimately, of your actions is being accountable. Responsibility is an obligation to perform work duties, or tasks using sound professional judgment. The nurse is accountable for the duties that they perform as a nurse and this begins by being accountable as a nursing student. As you prepare for a career in nursing you are accountable to yourself, your peers, your nursing faculty, and the clients assigned to your care. Ways that you can demonstrate accountability and responsibility as a nursing student include being on time to class and clinicals, preparing for class and clinical assignments, completing assignments to the best of your ability and turning them in on time, practicing skills in the laboratory setting prior to performing them with clients, and practicing within the scope of practice for a nursing student.

A nurse is delivering client care in partnership with multidisciplinary members of the healthcare team. This is an example of which of the following QSEN competencies? Safety Patient-centered care Quality improvement Teamwork and collaboration

Teamwork and collaboration

A nurse is caring for a client who is scheduled for cardiac surgery and tells the nurse, "I don't think I'm going to have the surgery. Everybody has to die sometime." Which of the following responses by the nurse is appropriate? 1. Clients having this surgery are always scared 2. Why have you changed your mind about the surgery 3. You shouldn't worry, everything will be fine 4. Tell me more about your concerns

Tell me more about your concerns

A nurse is observing an assistive personnel (AP) obtain vital signs from an adult client. Which of the following actions by the AP requires follow up by the nurse? The AP pulls the pinna up and back when obtaining a tympanic temperature. The AP informs the client when they are counting the respirations. The AP gently presses down with the pads of two to three fingers over the radial pulse site. The AP selects a blood pressure cuff width that is 40% the circumference of the client's arm.

The AP informs the client when they are counting the respirations.

A nurse is observing an assistive personnel (AP) who is obtaining a blood pressure reading from a client. Which of the following actions by the AP requires follow up by the nurse? The AP uses a cuff width that is 40% of the circumference of the client's arm. The AP provides support for the client's arm while taking the BP. The AP waits to take the client's BP 45 min after the client ambulates in the hallway. The AP loosens the valve to reduce pressure within the bladder cuff at a rate of 5 mm Hg per second.

The AP loosens the valve to reduce pressure within the bladder cuff at a rate of 5 mm Hg per second.

A nurse resides in a state that recently enacted the nurse licensure compact. Which of the following information should the nurse identify as a part of the NLC? The NLC requires a nurse to obtain a license for each state in which they wish to practice. The NLC allows a nurse to practice across state lines with one license. The NLC allows a nurse to have a suspended license when practicing under a multistate license. The NLC is not a good option for telehealth nursing.

The NLC allows a nurse to practice across state lines with one license.

A nurse is educating a group of nurses about the nurse practice act. Which of the following statements should the nurse include? (select all that apply) The NPA contains current laws and regulations for nursing practice The NPA is the same for all states The NPA contains standards and scope of practice The federal government enforces the NPA for each state The nurse is responsible to know current roles and responsibilities as defined by the NPA

The NPA contains current laws and regulations for nursing practice The NPA contains standards and scope of practice The nurse is responsible to know current roles and responsibilities as defined by the NPA

In an urgent care setting, the PN is caring for a client who reports nausea. When collecting data, which of the following questions should the PN ask the client regarding the nausea?

The PN should collect data relating to nausea. When did the nausea begin? What did you have to eat or drink, if anything, before the nausea began? Has the nausea been constant or intermittent? Have you done anything or taken any medication for your nausea? Did it help? Have you vomited? If so, how many times? When did the vomiting begin?

The Standards of Professional Performance describe those behaviors expected of a nurse, including quality of practice. You are asked to perform a procedure with which you are unfamiliar, but that is within your scope of practice. What would you do?

The Standards of Professional Performance state that nurses evaluate their own nursing practice. Nurses should seek out information and guidance to ensure they are providing competent care that reflects current practice. Nurses are responsible for their professional actions to those with whom they interact professionally, including clients, colleagues, and society.

A nurse is caring for a client who is postoperative and has sanguineous drainage on their dressing. The nurse is unable to reach the client's provider. Which of the following members of the chain of command should the nurse contact next? The charge nurse The nursing supervisor The divisional director The chief medical officer

The charge nurse

A nurse is caring for a postoperative client who states that he is worried about being discharged after surgery because he has no place to live. Describe how the biophysical model of pain, particularly the social factors, contributes to this client's experience of pain.

The client is expressing concern that he has nowhere to live upon discharge. According to the biophysical model of pain, the client likely does not have anyone to care for him after discharge and may lack a social support system, which can lead to increased anxiety and perception of pain. Economic factors that may affect the client include that, if the client has no home, the client may not be employed, have insurance, or be able to pay for follow-up care, prescriptions, supplies, or therapies. These concerns can contribute to an increase in the client's pain level and response to stimuli.

A nurse is performing an admission assessment on a client. Using the safety and risk reduction priority setting framework, which of the following findings should the nurse identify as the priority? The client reports dizziness when standing Client has not had a bowel movement in 3 days Client has non-pitting edema in the lower extremities Has several scratch marks on their abdomen

The client reports dizziness when standing

A newly licensed nurse is reviewing electronic information about the NCLEX exam from the national Council of state board of nursing website. Which of the following information should the nurse identify as passing criteria for the NCLEX computerized adaptive test? The computer will stop giving items using the 95% confidence interval rule The board of nursing for each state determines the passing grade for the NCLEX exam The candidate receives the maximum number of items available The candidate runs out of time before answering the minimum number of items

The computer will stop giving items using the 95% confidence interval rule

A PN is contributing to the plan of care for a client who has nausea. After collaborating with the RN, a plan is developed that calls for administering a prescribed promethazine suppository for the client's nausea. What might an appropriate client-centered goal for this plan include?

The goal is that the client will report a decreased nausea level and the absence of vomiting within an hour of administration of the promethazine suppository.

A newly licensed nurse is reviewing the standards of care when performing client care. Which of the following describes a standard of care? Explanatory statements that describe the competent level of care for all nurses to follow in practice A component of the nursing scope of practice that specifies the competent level of behavior expected in the professional role Part of the nursing scope of practice that specifies the nurse's role in alleviating client suffering and promoting client healing The level of care a nurse provides that is the same for all nurses who have comparable education, competency, and experience

The level of care a nurse provides that is the same for all nurses who have comparable education, competency, and experience

A nurse is discussing professional commitment with their supervisor during an annual performance review. Which of the following actions by the nurse demonstrates professional commitment? The nurse attends an educational course to learn about the treatment and prevention of pressure ulcers. The nurse tries to improve their skills by starting an unnecessary IV on a client who has dementia. The nurse covers for another nurse who the nurse suspects is chemically impaired. The nurse withholds information from a client to reduce the client's stress level.

The nurse attends an educational course to learn about the treatment and prevention of pressure ulcers.

Other than the Numeric Rating Scale, when assessing pain in a client who speaks another language than the nurse, what alternative scale can the nurse use?

The nurse can use the Wong-Baker FACES Pain Rating scale. The client is asked to choose the face that represents how they feel. In face 0, the client has no pain; in face 10, the client has the worst pain they can imagine. Other faces are scored as 2, 4, 6, and 8, with each indicating more pain than the previous face.

A client was admitted to a medical-surgical unit at 0900. The client reported experiencing abdominal pain during the night, which was unrelieved after taking oral pain medication prescribed by the provider. A family member transported the client to the emergency department (ED). The client received IV morphine in the ED and was admitted to the medical-surgical unit for further workup of the abdominal pain. After completing an initial admission assessment, the nurse left the room to discuss the client's plan of care with the attending provider. Upon returning to the client's room several minutes later, the nurse observes that a visitor is with the client. The visitor pulls the nurse aside, stating, "There is something wrong with my friend. She keeps referring to me as her brother, but her brother passed away many years ago. She also thinks that I have come to visit her at her home." How should the nurse categorize the client's needs at this time using urgent versus nonurgent categorization?

The nurse should categorize a change in neurologic status as an urgent need. Clients who are experiencing respiratory difficulty, chest pain, or a change in neurologic status are prioritized as urgent. The nurse should collect further client data and report this finding.

A nurse caring for a client is unable to reach the provider to inform them of the client's low blood pressure. The nurse documents the efforts to make contact. What should the nurse do next?

The nurse should continue to move up the chain of command until they get the appropriate response. The nurse is required to document their efforts as they move along the chain of command. If one link in the chain fails, then the nurse continues to communicate and document to the next link and to the next link until they get the response needed.

a nurse is reinforcing teaching about transdermal nitroglycerin to a client who has stable angina. which of the following statements by the client indicates teaching has been effective I should leave the patch on for 16 to 20 hours each day I will apply a new patch in the same location each day The path should be effective within an hour of being applied The medication is not absorbed as well when placed on the abdomen

The path should be effective within an hour of being applied

All licensed nurses must follow the state-mandated scope of practice. What guidance does the scope of practice provide for nurses?

The scope of practice defines those duties that a nurse is considered competent to carry out and authorized to perform as specified by state law. In other words, the scope of practice stipulates those activities nurses can and cannot perform legally.

A nurse is providing education on priority setting frameworks to a group of newly licensed nurses. Which of the following statements should the nurse make regarding the safety and risk reduction priority setting framework? When using this framework, clients are prioritized using a color-coded system This framework uses the least restrictive measures first as long as the client's safety is maintained When using this framework, the nurse will encourafge the client to have social relationships through social interactions This framework assigns the highest priority to the situation that poses a threat to the client's physical well-being

This framework assigns the highest priority to the situation that poses a threat to the client's physical well-being

A nurse is reviewing the vital signs obtained by an assistive personnel at 1200. For which of the following clients should the nurse plan to intervene? Adult male who has a respiratory rate of 18/min Toddler who has a respiratory rate of 44/min Newborn who has a respiratory rate of 56/min Adolescent female who has a respiratory rate of 16/min

Toddler who has a respiratory rate of 44/min

Give an example/describe the different types of leaders: transactional, transformational, laissez-faire, bureaucratic, situational

Transactional: if we beat out the other units with a 100% compliance, we will pass out pen lights and scissors Transformational: together we can be the best and first unit to actively involve the client in bedside report Laissez-faire: evidence-based research shows that bedside reporting is best. Come up with a way to start this Bureaucratic: this checklist is the process to be used for bedside reporting. It will be started on the 15th of the month Situational: tell me what you know about bedside reporting along with what you think about it

A nurse is caring for a client who reports a new onset of abdominal pain. The nurse should assign the client's condition to which of the following categories when prioritizing care? Urgent Chronic Minimal Expectant

Urgent

A nurse is collecting data on a client who has received a preoperative dose of morphine. Give an example of an adverse effect of the medication

Urinary retention

A nurse is discussing the use of a client's thigh for blood pressure measurements with an assistive personnel (AP). Which of the following information should the nurse include? Select a blood pressure cuff width that is 25% of the circumference of the client's thigh. Palpate the femoral pulse when obtaining blood pressure in the thigh. Expect blood pressure in the thigh to be 10 to 15 mm Hg less than in the arm. Use the thigh to obtain blood pressure when a client has severe edema in their arms.

Use the thigh to obtain blood pressure when a client has severe edema in their arms.

A graduate nurse is offered a position in the emergency department upon the granting of initial licensure. Which of the following information should the graduate nurse identify as a requirement during the initial licensure process? (select all that apply) Verification of graduation Criminal background check in certain states Passing score on NCLEX examination Submitting continuing education hours with the application Self-disclosure of any substance use in the past 5 years

Verification of graduation Criminal background check in certain states Passing score on NCLEX examination Self-disclosure of any substance use in the past 5 years

Due to increased shortness of breath, an older adult client was brought to an urgent-care clinic by their adult child. The child reported that the client had become increasingly short of breath over the previous two days. The client has a history of anxiety, which is managed with PRN lorazepam, and has fallen two times within the past week. Their child reports that the client has been upset recently at the loss of their best friend, whom they had known for many years. The client's vital signs are as follows: BP 130/84 mm Hg, pulse 88/min, respiratory rate 24/min, oxygen saturation 90% on room air. The client's adult child asks the nurse what their plan of action is in caring for the client. What should the nurse identify as this client's priority problem?

When using Maslow's hierarchy of needs, the nurse should identify the client's physiological needs as the priority. This client is experiencing increased shortness of breath, and his respiratory rate and oxygen saturation levels are above the expected range. The nurse should administer oxygen to the client.

A nurse in a rehabilitation facility has received report on four clients. Which of the following should the nurse evaluate first? A) A client who has peripheral vascular disease and reports numbness in the toes B) client who has depression & is easily distracted C) A client who has Alzheimer's disease and is unable to complete ADLS D) A client who had abdominal surgery 10 days ago and reports feeling his incision pop

a client who had abdominal surgery 10 days ago and reports feeling his incision pop

A nurse is reviewing information for several clients on the unit. the nurse should recognize that which of the following clients is at greatest risk for Respiratory depression? a client who has chronic back pain and recently started taking paroxetine a client who has cancer and has taken oxycodone PRN for several months a client who has been accidentally taking twice the amount of Prednisone as prescribed a client who had surgery 3 hours ago and is receiving IV hydromorphone PRN

a client who had surgery 3 hours ago and is receiving IV hydromorphone PRN

A nurse is evaluating a group of clients who are experiencing pain. which of the following clients should the nurse identify as experiencing neuropathic pain? a client who has osteoarthritis and reports difficulty ambulating for the past 6 months a client who had surgery to repair a fractured tibia and reports incisional pain a client who has diabetes mellitus and reports bilateral burning foot pain without signs of injury a hospice client who has prostate cancer and reports pelvic pain

a client who has diabetes mellitus and reports bilateral burning foot pain without signs of injury

A nurse in a provider's office has collected data on four clients. Which of the following clients should be the nurse's priority concern? A) a client who is has a history of HF B) a client who has type 1 DM C) a client who is reporting pain associated w/ osteoarthritis of the knees D) a client who is having a nosebleed associated w/ hypertension

a client who is having a nosebleed associated w/ hypertension

Nurses assisting with a staff in service regarding pain control. which of the following statements by a staff member indicates an understanding of the information? ( select all that apply) a client's religious beliefs might affect the way they respond to pain herbal therapies are not permitted for a client receiving prescription pain medication the clients passed pain experiences or not related to their current pain and Pain Management if a client can rate their pain using a numeric pain scale, there is no need to know nonverbal findings pain control might be hard to achieve if the nurse and client speak different primary languages

a client's religious beliefs might affect the way they respond to pain the clients passed pain experiences or not related to their current pain and Pain Management pain control might be hard to achieve if the nurse and client speak different primary languages

A nurse is planning to teach coworkers about the legal and ethical principles used with pain management. which of the following examples should the nurse include as an example of autonomy? the nurse allows a client to wait longer for their pain medication than other clients a nurse does not properly clean a vile of pain medication prior to withdrawing medication from the vile, which results in the client Contracting an infection a nurse administer is scheduled pain medication and provides therapeutic distraction techniques for a client in pain a nurse provides a client with the opportunity to take an intramuscular injection or oral medication for pain relief

a nurse provides a client with the opportunity to take an intramuscular injection or oral medication for pain relief

A nurse is caring for a client who has a prescription for hydromorphone 1 to 2 mg I am every 4 hours is needed for a pain rating of four to six on a 0 to 10 scale. the client has never taken Hydromorphone before. which of the following actions should the nurse plan to take? administer 1 mg IM request a prescription to give the medication IV instead request a prescription for a different medication administer 2 mg IM

administer 1 mg IM

A nurse is talking w/ a client who arrived at the clinic over an hour ago and states, "doesn't anyone care that I am sick? Why do I have to wait so long?" The nurse listens to the needs of the client. In which of the following roles is the nurse performing? - educator - case manager - advocate - leader

advocate

A nurse is preparing to obtain a young adult client's apical pulse. In which of the following locations should the nurse place their stethoscope to auscultate the client's pulse? apex of the heart Right side of sternum 4th intercostal space Midclavicular line below right clavicle

apex of the heart

A nurse is measuring the blood pressure of a client who has a fractured femur. the blood pressure reading is 140/94, and the client denies any history of hypertension. which of the following actions should the nurse take first? request a prescription for any hypertensive medication ask the client if they are having pain request a prescription for an anti-anxiety medication return in 30 minutes to recheck the client's blood

ask the client if they are having pain

A nurse observes an assistive personnel reprimanding a client for not using the urinal properly. The AP tells the client that diapers will be used next time the urinal is used improperly. Which of the following torts is the AP committing? Assault Battery False imprisonment Invasion of privacy

assault Threatening the client = assault

A charge nurse is observing a newly licensed nurse care for a client who reports pain. the nurse checked the client's MAR and noted the last dose of pain medication was 6 hours ago. the prescription reads every 4 hours PRN for pain. the nurse administered the medication and checked the client 40 minutes later, when the client reported improvement. the newly licensed nurse left out which of the following steps for the nursing process? assessment planning intervention evaluation

assessment

a nurse is caring for a client who has cancer. the client has decided to stop treatment and requests a referral to hospice. by making the referral as requested, the nurse is illustrating which of the following ethical principles. Justice Autonomy Veracity Fidelity

autonomy

A nurse is caring for a client who has been prescribed an indwelling urinary catheter. When preparing to insert the catheter, the nurse should first open the sterile package in which of the following directions? 1. to the left 2. to the right 3. away from the body 4. toward the body

away from the body

A nurse is caring for a client who is 24 hour postoperative following an inguinal hernia repair. The client is tolerating clear liquids well, has active bowel sounds, and is expressing a desire for "real food". The nurse tells the client, "I will call the surgeon and ask for a change in diet." The surgeon here's the nurses report and prescribes a full liquid diet. The nurse used which of the following levels of critical thinking? Basic Commitment Complex Integrity

basic

A nurse is evaluating a client's pain level using the PQRST mnemonic. which of the following questions should the nurse ask to evaluate the letter r? Can you rate your pain on a scale of 0 to 10, with zero being no pain and 10 being the worst pain you can imagine? can you point to where you are having pain what does your pain feel like? what were you doing when your pain started?

can you point to where you are having pain

A nurse is caring for a client who is diagnosed with gastroenteritis. Which of the following actions should the nurse take first when evaluating for fluid volume deficit? A) obtain an arterial pH level B) check the HR and BP C) insert an indwelling catheter D) collect a serum BUN and creatinine

check the HR and BP

A nurse is assisting with the admission of a client who has decreased circulation in the left leg. Which of the following is the first action the nurse should take? A) Administer an anticoagulant B) Check the leg for warmth and Edema C) Apply elastic stockings D) Promote bed rest & extremity elevation

check the leg for warmth and Edema

A nurse is caring for a client who is prescribed IV fluids. While inserting the IV catheter, blood spilled on the floor. Which of the following solutions should the nurse use to clean the spill? 1. isopropyl alcohol 2. chlorhexidine gluconate (hibiclens) 3. chlorine (bleach) 4. Iodophor

chlorine (bleach)

A nurse is caring for a client who reports muscle pain to the lower back that has persisted for over a year after motor vehicle crash. in which way should the nurse categorize this client's pain? cancer pain acute pain Chronic pain neuropathic pain

chronic pain

A newly licensed nurse is reviewing Benner's Novice to Expert Model for nursing competence. At which of the following stages does the nurse first develop the ability to prioritize tasks by drawing on experience? - advanced beginner - proficient -competent - novice

competent

a nurse is caring for a client who has a history of falls. which of the following actions is a nurse's priority? complete a full risk assessment educate the client and family about fall risks eliminate safety hazards from the client's environment make sure the client uses assistive aids in their possession

complete a full risk assessment

A nurse is reviewing the plan of care for several clients who are receiving treatment for pain. which of the following actions should the nurse plan to take to evaluate the client's pain control? (select all that apply) consider each client's cultural preferences determine the effectiveness of non-pharmacological strategies record the client subjective reports rather than the nurses objective observations recognize that older adult clients over report use a Pain Scale specific to each client's cognitive abilities

consider each client's cultural preferences determine the effectiveness of non-pharmacological strategies use a Pain Scale specific to each client's cognitive abilities

A nurses caring for an older adult client who recently experienced the death of her partner. Which of the following is the priority need of the client? A) establishing a sense of achievement B) contributing to society C) creating meaningful social relationships D) enhancing self- confidence

creating meaningful social relationships

A nurse in a long-term care facility is assisting with the admission of several clients. To prevent falls in hospitalized clients, which of the following actions should the nurse take first? A) Provide assistance w/ ambulation when indicated B) determine the mobility status of each patient C) Maintain the side rails of each be in the raised position D) Plan a fall prevention program for clients at risk

determine the mobility status of each patient

A nurse is collecting data on four clients. Which of the following is the highest priority finding by the nurse? A) Malaise B) Anorexia C) Headache D) Diarrhea

diarrhea

A nurse is conducting a breast examination on a client who has a family history of breast cancer. Which of the following should the nurse report tot he provider? 1. silver-colored striae 2. unilateral nipple inversion present since menarche 3. dimpling of the tissue in the upper outer quadrant 4. visible symmetrical venous patterns

dimpling of the tissue in the upper outer quadrant

A nurse uses a head to toe approach to conduct a physical assessment of a client who will undergo surgery the following week. which of the following critical thinking attitudes did the nurse demonstrate? confidence perseverance Integrity discipline

discipline

A nurse preceptor is orienting a newly licensed nurse. Which of the following actions by the newly licensed nurse indicates a breach of confidentiality and requires intervention by the nurse preceptor? 1. faxing lab results to a client's provider 2. discussing changes in a client's plan of care with his friend who is a nurse on another unit 3. describing a client's level of independence to the case manager arranging home health services 4. remaining in the room with the client while he reviews his own medical records

discussing changes in a client's plan of care with his friend who is a nurse on another unit

a nurse is instructing an assistive Personnel about caring for a client who has a low platelet count. which of the following instructions is the priority for measuring vital signs for this client? do not measure the client's temperature rectally count the client's radio pulse for 30 seconds and multiply it by 2 do not let the client know you are counting respirations let the client rest for 5 minutes before you measure their blood pressure

do not measure the client's temperature rectally

A nurse is caring for an older adult client who has cognitive impairment and is postoperative. which of the following actions should the nurse take? use the crying, requires oxygen, increase vital signs, expression, sleeplessness CRIES Pain Scale reassure family members that older adult clients have a decreased ability to sense pain evaluate the client for pain by observing their behavior assign a Pain Scale number based on the FACES Pain Scale

evaluate the client for pain by observing their behavior

A nurse is discussing end of life pain management with a group of coworkers. Which of the following should the nurse include as barriers to end of life pain management? ( select all that apply) fear of addiction belief that pain is an expected part of their illness inability to sleep lack of support inadequate pain assessment

fear of addiction belief that pain is an expected part of their illness inadequate pain assessment

A newly licensed nurse is considering strategies to improve critical thinking. which of the following actions should the nurse take? ( select all that apply) find a mentor use a journal to write about the outcomes of clinical judgments review articles about evidence-based practice. limit consultations with other professionals involved in a client's care make quick decisions when unsure about a client's needs.

find a mentor use a journal to write about the outcomes of clinical judgments review articles about evidence-based practice.

A nurse is preparing to administer oral medication to a client who has unilateral weakness following a cerebrovascular accident (CVA). Which of the following should be the priority action of the nurse? A) Administer medications w/ meals when possible B) Ensure client understanding of medication's effects C) Determine the client's ability to self-administer meds D) have the client position the head w/ chin down while swallowing

have the client position the head w/ chin down while swallowing

A nurse is caring for a client who tells the nurse they want to quit smoking. The nurse provides the client with information about smoking cessation and other ways to improve their health. Which of the following nursing roles is the nurse demonstrating? (Select all that apply) - Advocate - collaborator - change manager - health promotion - nurse educator

health promotion nurse educator

A nurse is caring for a newly admitted client. Which of the following client needs should the nurse address first? A) homelessness B) lack of family support C) Hypoxic D) under nourished

hypoxic

A newly hired nurse is reviewing the facilities emergency preparedness plan. Based on a review of the four triage categories, the nurse should provide power you care to clients who are in which of the following categories during a disaster? A) immediate B) delayed C) minimal D) expectant

immediate

A nurse is caring for a client who has a new prescription for antihypertensive medication. prior to administering the medication, the nurse uses an electronic database to gather information about the medication and the effects it might have on this client. which of the following components of critical thinking is a nurse using when he reviews the medication information? knowledge experience intuition competence

knowledge

A nurses caring for a client who is in the immediate post operative period following a tracheotomy. Which of the following is the nurses priority action? A) providing pain control B) preventing hemorrhage C) maintaining a patent airway D) ensuring adequate fluid intake

maintaining a patent airway

A nurse is caring for a client who fell at a nursing home. the client is oriented to person, place, and time and can follow directions. which of the following actions should the nurse take to decrease the risk of another fall? ( select all that apply) place a belt restraint on the client when they are sitting on the bedside commode keep the bed in its lowest position with all the side rails up make sure the clients call light is Within Reach provide the client with non-skid Footwear complete a fall risk assessment

make sure the clients call light is Within Reach provide the client with non-skid Footwear complete a fall risk assessment

a nurse observes smoke coming from under the door of the staff's lounge. which of the following actions is a nurse's priority? extinguish the fire activate the fire alarm move clients who are nearby close all open doors on the unit

move clients who are nearby

A nurse is discussing the use of heat therapy with a newly licensed nurse period the nurse should include that heat therapy is effective for which of the following conditions? ( select all that apply) muscular pain active bleeding backache menstrual discomfort swollen extremity

muscular pain backache menstrual discomfort

A nurse is assessing a client using the ABCDE priority setting approach. which of the following actions should the nurse take when completing the exposure component of this priority setting method? ( select all that apply) observe the clients lower extremities for indications of DVT obtain a respiratory rate for one full minute measure the client's temperature check the client for bruising obtain a blood pressure measurement

observe the clients lower extremities for indications of DVT measure the client's temperature check the client for bruising

A nurse is caring for a client who has a serum potassium level of 3.1 mEq/L. Which of the following actions should the nurse take first? A) Obtain an ECG. B) Administer oral potassium C) Encourage potassium-rich foods D) Monitor I & O

obtain an ECG.

A nurse is caring for a client in the emergency department who has an oral body temperature of 101, pulse rate 114/minute, and respiratory rate 22/minute. the client is restless with warm skin. which of the following interventions should the nurse take? ( select all that apply) obtain culture specimens before initiating antimicrobials restrict the client's oral fluid intake encourage the client to rest and limit activity allow the client to shiver to dispel excess assist the client with oral hygiene frequently

obtain culture specimens before initiating antimicrobials encourage the client to rest and limit activity assist the client with oral hygiene frequently

A nurse is collecting data on a recently admitted client. Which of the following techniques should the nurse use to measure tissue perfusion 1. determining the client's respiratory rate 2. measuring the client's chest diameter 3. obtaining the client's level of oxygen saturation 4. checking the client's depth of respirations

obtaining the client's level of oxygen saturation

The nurse is caring for a client who has a prescription for Oxycodone 5 to 10 mg PO every 4 to 6 hours is needed for pain rating 7 to 10 on a 0 to 10 scale. 15 minutes after receiving the dose, the client reports to their nurse their pain is still a 7 and has not changed. which of the following actions should the nurse take? administer another 5 mg dose of the oral opioid now administer 10 mg of oxycodone every 2 hours inform the provider that the client's pain medication is not effective offer to assist the client with non-pharmacological relief strategies

offer to assist the client with non-pharmacological relief strategies

A nurse is caring for a client who is diagnosed with anemia. Which of the following skin color variations is caused by a reduced amount of oxyhemoglobin? cyanosis 2. Jaundice 3. Erythema 4. pallor

pallor

a nurse's instructing a group of assistant personnel in measuring a client's respiratory rate. which of the following guidelines should the nurse include? ( select all that apply) place the client in semi Fowler's position have the client rest and arm across the abdomen observe one full respiratory cycle before counting the rate count the rate for 30 seconds if it is irregular count and Report any sighs the client demonstrates

place the client in semi Fowler's position have the client rest and arm across the abdomen observe one full respiratory cycle before counting the rate

A charge nurse is assigning client care for four clients. which of the following tasks should the nurse assign to a PN? creating a plan of care for a client who is recovering following a stroke. assessing a pressure injury on a client who is on bed rest.l providing nasopharyngeal suctioning for a client who has pneumonia. teaching a client who has asthma to use a metered dose inhaler

providing nasopharyngeal suctioning for a client who has pneumonia.

When entering a client's room to change a surgical dressing, a nurse notes that the client is coughing and sneezing. which of the following actions should the nurse take when preparing the sterile field? keep the sterile field at least 6 ft away from the client's bedside instruct the client to refrain from coughing and sneezing during the dressing change put a mask on the client to limit the spread of microorganisms into the surgical wound keep a box of facial tissues nearby for the client to use during the dressing change

put a mask on the client to limit the spread of microorganisms into the surgical wound

Which of the following legal guidelines should be followed when documenting in a client's record? (select all that apply) cover errors with correction fluid and right in the correct information put the date and time on all entries document objective data, leaving out opinions use as many abbreviations as possible wait until the end of the shift to document

put the date and time on all entries document objective data, leaving out opinions

A nurse is precepting a newly licensed nurse while he is charting. Use of which of the following abbreviations indicates a need for further teaching 1. Mcg 2. q.d. 3. mL 4. PO

q.d.

A nurse On a medical - surgical unit has received change Of shift report and will care for four clients. which of the following tasks should the nurse assign to an assistive personnel? updating the plan of care for a client who is post-operative. reinforcing teaching with a client who is learning to walk using a quad cane. reapplying a condom catheter for a client who has urinary incontinence. applying a sterile dressing to a pressure injury

reapplying a condom catheter for a client who has urinary incontinence.

By the second post-operative day, a client has not achieved satisfactory pain relief. based on this evaluation, which of the following actions should the nurse take, according to the nursing process? reassess the client to determine the reasons for inadequate pain relief. wait to see whether the pain lessons during the next 24 hours. change the plan of care to provide different pain relief interventions. teach the client about the plan of care for managing the pain

reassess the client to determine the reasons for inadequate pain relief.

A nurse manager is assigning care of a client who is being admitted from the PACU following thoracic surgery. the nurse manager should assign the client to which of the following staff members? charge nurse. registered nurse practical nurse. assistive personnel

registered nurse

A nurse is preparing an in-service program about delegation. which of the following are components of the five rights of delegation? ( select all that apply.) right place right supervision and evaluation right direction and communication right documentation right circumstances

right supervision and evaluation right direction and communication right circumstances

A nurse is caring for a male client who has been prescribed an indwelling urinary catheter. In which of the following positions should the client be placed for insertion of the catheter? 1. dorsal recumbent 2. Orthopneic 3. Side-lying 4. supine

supine

a nurse is delegating the ambulation of a client who had knee orthoplasty five days ago to an AP. which of the following information should the nurse share with the AP? (select all that apply). the roommate ambulates independently. the client ambulates wearing slippers over anti-embolic socks. the client uses a front-wheeled walker when ambulating the client had pain medication 30 minutes ago. the client is allergic to codeine. the client ate 50% of breakfast this morning

the client ambulates wearing slippers over anti-embolic socks. the client uses a front-wheeled walker when ambulating the client had pain medication 30 minutes ago.

A nurse is caring for a client who has kidney stones. which of the following manifestations is an objective indicator of pain? the client is diaphoretic the client is experiencing stabbing pain the client is nauseated the client states feeling dizzy

the client is diaphoretic

a nurse has removed a sterile pack from its outside cover and placed it on a clean work surface in preparation for an invasive procedure. which of the following flaps should the nurse unfold first? black closest to the body the side flap the left side flap the flap furthest from the body

the flap furthest from the body

A nurse is wearing sterile gloves in preparation for performing a sterile procedure period which of the following objects can the nurse touch without breaching sterile technique? ( select all that apply) a bottle containing a sterile solution the edge of the sterile drape at the base of the field the inner wrapping of an item on the sterile field an irrigation syringe on the sterile field one gloved hand with the other gloved hand

the inner wrapping of an item on the sterile field an irrigation syringe on the sterile field one gloved hand with the other gloved hand

a nurse has prepared a sterile field for assisting a provider with a chest tube insertion. which of the following events should the nurse recognize as contaminating the sterile field? (select all that apply) the provider drops a sterile instrument onto the near side of the sterile field the nurse moistens a cotton ball with sterile normal saline and places it on the sterile field the procedure is delayed one hour because the provider receives an emergency call the nurse turns to speak to someone who enters through the door behind the nurse the client's hand brushes against the outer edge of the sterile field

the nurse moistens a cotton ball with sterile normal saline and places it on the sterile field the procedure is delayed one hour because the provider receives an emergency call the nurse turns to speak to someone who enters through the door behind the nurse

A nurse is discussing cutaneous stimulation with a client who has back pain. which of the following methods should the nurse include? ( select all that apply) transcutaneous electronic stimulating unit (TENS unit) now distraction techniques Massage Acupuncture cold therapy

transcutaneous electronic stimulating unit (TENS unit) now Massage Acupuncture cold therapy

a 38 year old client who has a fever due to an infection is at risk for experiencing tachycardia. True or false

true

A nurse preceptor is working with a newly licensed nurse to transfer a client from the bed to a chair. Which of the following actions by the new nurse indicates a need for further teaching to prevent lift injuries 1. twisting at the waist and shoulders 2. standing with feet in a wide stance 3. positioning self close to the client 4. using arms and legs to lift

twisting at the waist and shoulders

A nurse is collecting data on four clients. Which of the following findings is the most urgent? A) bladder distension and urgency B) pedal edema C) warmth and pain in the calf D) hypoactive bowel sounds

warmth and pain in the calf

a nurse is reviewing hand hygiene techniques with a group of assistive personnel. which of the following instructions should the nurse include when discussing hand-washing? ( select all that apply) apply 3 to 5 ml of liquid soap to dry hands wash the hands with soap and water for at least 15 seconds Rinse the hands with hot water use a clean paper towel to turn off hand faucets allow the hands to air dry after

wash the hands with soap and water for at least 15 seconds use a clean paper towel to turn off hand faucets

A nurse is discussing transcutaneous electrical nerve stimulation TENS treatment with a client who has chronic lower back pain. which of the following statements should the nurse include? ( select all that apply) you can be taught how to use TENS therapy at home we will insert very small sterile needles into your skin to block your pain this therapy may result in you having some temporary bruising at the site of application the TENS therapy delivers low voltage electrical impulses to the skin over painful areas we will adjust the intensity, pulse rate, and duration of the electrical pulses during your therapy

you can be taught how to use TENS therapy at home the TENS therapy delivers low voltage electrical impulses to the skin over painful areas we will adjust the intensity, pulse rate, and duration of the electrical pulses during your therapy

A nurse is providing end of life care for a patient who is unresponsive and near death. the client's family asks the nurse about managing the client's pain. which of the following statements should the nurse make to the client's family? your family member will not require pain medication your family member can inform the provider about their decision for pain management your family member has the right to receive effective pain management your family member will not be able to tolerate the effects of pain medication

your family member has the right to receive effective pain management


Ensembles d'études connexes

21.9 - Regulation of mean arterial pressure

View Set

Abeka Consumer Mathematics Quiz 5

View Set

Patient Assessment paramedic hocking

View Set

Monopolistic Competition and Oligopoly

View Set

Björn's new examination (utom Le Guin och Vonnegut)

View Set

Resistance exercise for impaired muscle performance

View Set

Aesthetics & Revolution : Futurism - The Avant Garde

View Set